You are on page 1of 86

MATHEMATICS

Class - XII Booklet - 2

Content

S.No. Chapter Name Page No.

1. Determinant 1 - 41
2. Matrix 42 - 85
IIT ASHRAM DETERMINANTS

DETERMINANT 1
UNIT

 Theory

 Drill Exercises

 Solved Examples

 Exercises

 Single Correct Type Questions

 Multiple Correct Type Questions

 Comprehension Type Questions

 Matrix Match Type Questions.

 Integer Type Questions

 Previous Years Questions

 Answer Key
DETERMINANTS

Determinant of order 2

a1 b1
The symbol a b2 is called the determinant of order two .
2

Its value is given by : D = a 1 b2 - a 2 b1


Determinant of order 3

a1 b1 c1
The symbol a 2 b2 c2 is called the determinant of order three
a3 b3 c3

Minors :
The minor of a given element of a determinant is the determinant of the
elements which remain after deleting the row & the column in which the
given element stands .
Cofactor :
If Mij represents the minor of some typical element then the cofactor is defined as :
Cij = (-1)i+j . Mij ; Where i & j denotes the row & column in which the particular
element lies.

1 2 3
Ex.-1: Find the Co-factors of all the elements of 4 5 6
7 0 1
5 6
Sol. (i) Minor of 1   5 , Co-factor of 1 = (–1)1 + 15 = 5
0 1

4 6
(ii) Minor of 2 =  4  42  46 , Co-factor of 2 = (–1)1 + 22 (–46) = 46
7 1

4 5
(iii) Minor of 3   35 , Co-factor of 3 = (–1)1 + 335 = 35
7 0

2 3
(iv) Minor of 4 =  2 , Co-factor of 4 = (–1)2 + 1(–2) = 2
0 1

1 3
(v) Minor of –5 =  22 , Co-factor of –5 = (–1)2 + 2(–22) = –22
7 1

Similarly the other Co-factors can be found.


Evaluation/Expansion of Determinant : Co-Factor Method
Sum of product of all the elements of a particular row (or column) with their
respective co-factors gives value of determinants

Page # 2
DETERMINANTS

a1 b1 c1
e.g. a 2 b2 c2 expanding along C 1
a3 b3 c3

b2 c2 b1 c1 b1 c1
D = a1 b c - a2 b c + a3 b c
3 3 3 3 2 2

OR

expanding along R 1

b2 c2 a 2 c2 a2 b2
D = a1 b c - b1 a + c1 a b3 .......
3 3 3 c3 3

In this manner we can expand a determinant in 6 ways using elements of ;


R 1 , R 2 , R 3 or C 1 , C 2 , C 3 .
Note : Value of a determinant remains unique and does not depend on the
selection of row (or column) for expansion.
Ex.2: If  is one of the imaginary cube roots of unity, then then evaluate the

1 3 2
determinant  3 1 
2  1

1 1 2
Sol: Put  3 = 1 1 1  and open by R1 to get (1 –  2) + (1 –  ) = 3
2  1

Properties of Determinant:
P- 1 : The value of a determinant remains unaltered , if the rows & columns are
inter changed .

a1 b1 c1 a1 a 2 a 3
e.g. if a
 = 2 b2 c2  b1 b2 b3 =  '
a3 b3 c 3 c1 c2 c3

 &  ' are transpose of each other .


P- 2 : If any two rows (or columns) of a determinant be interchanged , the
valueof determinant is changed in sign only .
P- 3 : If a determinant has any two rows (or columns) identical , then its value
is zero.

a1 a1 c1
e.g., a2 a2 c2  0
a3 a3 c3

P- 4 : If all the elements of any row (or column) be multiplied by the same number,
then the determinant is multiplied by that number.

. Page # 3
DETERMINANTS

a1 kb1 c1 a1 b1 c1 a1 b1 c1 ka1 kb1 kc1


e.g., a kb c  k a b c and k a2 b2 c2  a 2 b2 c2
2 2 2 2 2 2

a 3 kb3 c3 a 3 b3 c3 a3 b3 c3 a3 b3 c3

a 5x p 3a 3b c
Ex.3: If b 10y 5 = 125, then x 2y z is equal to -
c 15z 15 p 5 5

3a 3b c 3a x p
Sol. x 2y z = 3b 2y 5 (changing rows into columns)
p 5 5 c z 5

3a x p a 5x p
1 3 1 1
= 3b 2y 5 = × b 10y 5 = (125) = 25.
3 3c 3z 15 3 5 c 15z 15 5

P-5 : If each element of any row (or column) can be expressed as a sum of two
terms then the determinant can be expressed as the sum of two
determinants.

a1 b1 c1  d1 a1 b1 c1 a1 b1 d1
a b2 c 2  d2  a 2 b2 c2  a 2 b2 d2
e. g. 2
a3 b3 c 3  d3 a3 b3 c3 a3 b3 d3

n n n

f(r) g(r) h(r)  f(r)


r 1
 g(r)
r 1
 h(r)
r 1
n
In general r 1
a 2 b2 c2  a2 b2 c2
a 3 b3 c3 a3 b3 c3

1 a a 2  bc
Ex. 4 Without expanding to any stage, prove that D = 1 b b2  ca = 0.
1 c c2  ab

1 a a 2 1 a bc
Sol. Splitting along third column D = 1 b b2  1 b ca = D1 -D2 (say)
1 c c2 1 c ab

a a 2 abc R1  aR1 


1  
D2 = b b2 abc R 2  bR 2 
abc R 3  cR 3 
c c2 abc

a a2 1 1 a a2
abc
= b b2 1  1 b b2 = D . Hence D = D -D = 0
abc 1 1 1
c c 2 1 1 c c2

. Page # 4
DETERMINANTS

r 1 n 6 n
Ex.5 If Dr = (r 1)2 2n2 4n  2 , then   r equals -
(r 1)3 3n2 3n2  3n r 1

n n(n 1)
Sol.   (r 1) = 1 + 2 +...+ (n – 1) =
r 1 2
n
2 n(n 1)(2n 1)
 (r 1) = 12 + 22 +...+ (n – 1)2 =
r 1 6
n
3 n2 (n 1)2
 (r  1) = 13 + 23 +...+ (n – 1)3 =
r 1 4

n(n 1)
n 6
2
1
n n(n 1)(2n 1) 2n2 2(2n 1)
   r = 6
r 1 1 2
n (n 1)2 3n3 3n(n  1)
4

6 n 6
n(n 1) 2(2n 1) 2n2 2(2n 1)
= = 0
12 3n(n 1) 3n3 3n(n 1)

P-6 : The value of a determinant is not altered by adding to the elements of


any row (or column) the same multiples of the corresponding elements of
any other row (or column).

Note : That while applying this property Atleast One Row (Or Column) must
remain unchanged.
P-7 : If by putting x = a the value of a determinant vanishes then (x - a) is a
factor of the determinant .
P-8 : If  ' = -  then it is Skew symmetric determinant but  ' = –   2  = 0
  = 0  Skew symmetric determinant of third order has the value zero

115 106 97
Ex.6 Show that D = 0 if D = 10 1 8 .
106 97 88

1
Sol. Operating C2  C2 – ( C1 + C3) , we get
2

115 0 97
D = 10 0 8 = 0
106 0 88

Note: Using the A.P. property one can immediately write D = 0 directly

. Page # 5
DETERMINANTS

13  3 2 5 5
Ex.7 Evaluate 15  26 5 10 .
3 65 15 5

13  3 2 1
Sol.   5. 5 15  26 5 2
3  65 3 5

13  3 2 1
R2  R2  2R1
 5 15  6 5 2 2 0 ,
R3  R3  5R1
3  15 3 2 5 0

15  6 5 2 2 5 2 5 2 2
5 5 3
3  15 3 2 5 3 5 3 2 5

5 3  5 2  3 2 5    3 5  5 2 2 
5 3  15  10  6  2 10  15  2 6  5  2 10 
5 3  6 5 
b2  c2 a2 a2
Ex.8 Evaluate the determinant b2 c2  a2 b2 .
2
c c2 a  b2
2

Sol. Applying R 1–(R 2 + R 3), we get

0 2c 2 2b2
Det. = b2 c2  a 2 b2
c2 c 2
a  b2
2

0 c2 b2
= 2 b2 c2  a 2 b2
c2 c 2
a  b2
2

0 c 2 b2
= 2 b2 a2 0 (by R2 + R 1, R 3 + R1)
c2 0 a2

= 2 (a2b2c2 + a 2b2c2)
= 4a 2b2c2

. Page # 6
DETERMINANTS

a  b  2c a b
Ex.9 Using properties prove that c b  c  2a b = 2(a + b + c)3
c a c  a  2b

Sol. Applying C 1 + C 2 + C 3, we get

1 a b
Det. = 2(a + b + c) 1 b  c  2a b
1 a c  a  2b

1 a b
= 2(a + b + c) 0 a  b  c 0 [by R 2 – R 1, R 3 – R 1]
0 0 a bc

= 2(a + b + c)3

x x2 1  x3
2 3
Ex.10 If x, y,z are different and y y 1  y  0 , then show that 1  xyz  0 .
2 3
z z 1 z
Sol. Writing the given determinant as the sum of two determinants, we have

x x2 1 x x2 x3
y y2 1 + y y2 y3 = 0
z z2 1 z z2 z3

x x2 1
 y y2 1 (1 + xyz) = 0
z z2 1

 (x – y) (y – z ) (z – x) (1 + xyz) = 0
 1 + xyz = 0 ( x  y  z)
 xyz + 1 = 0

n! (n  1)! (n  2)!
Ex.11 Prove that (n 1)! (n  2)! (n  3)! = (n!)3 ((2n3 + 8n2 + 10n + 4) for all (n  N)
(n  2)! (n  3)! (n  4)!

Sol. Here D = (n!)3

1 n 1  n  2 n  1
n 1  n  2 n  1  n  3 n  2 n  1
 n  2  n  1  n  3  n  2 n  1  n  4  n  3 n  2 n  1

1 1 1
= (n!)3 (n + 1)2 (n + 2) n 1 n2  n  3
 2 n  1
n   n  3  n  2  4  n  3 
n 

operating C2 – C 1, C 3 – C 2 and expanding = (n!)3 (n+1)2 (n + 2). 2


= (n!)3 ((2n3 + 8n2 + 10n + 4) as on simplification.

. Page # 7
DETERMINANTS
DRILL - I

2 4 3 
1. Find minors and cofactors of each element of second row of 1 6 4  and hence
5 0 2
 
evaluate its determinant.

a  b 2a  b 3a  b
2. Using properties of determinants, prove that 2a  b 3a  b 4a  b  0 .
4a  b 5a  b 6a  b

1 2
a bc
a
1 2
3. Using properties of determinantes, prove that b ac  0 .
b
1 2
c ab
c

a a2 bc 1 a2 a 3
4. Without expanding, prove that b b2 ca  1 b2 b3 .
c c2 ab 1 c2 c3

a  bx c  dx p  qx a c p
5. Without expanding, prove that ax  b cx  d px  q  1  x2  b d q .
u v w u v w

1 1 1
6. Prove that a b c   a  b b  c  c  a  a  b  c  .
a3 b3 c3

 a  1 a  2 a 2 1
7. Using properties of determinants, prove that  a  2 a  3 a  3 1  2 .
 a  3 a  4 a4 1

a2  2a 2a  1 1
3
8. Prove, using properties of determinant : 2a  1 a  2 1   a 1 .
3 3 1

a bc 2a 2a
3
9. Prove that 2b b c a 2b   a  b  c .
2c 2c c a b

3a a  b a  c
10. Prove that b  a 3b b  c  3  a  b  c  ab  bc  ca  .
c  a c  b 3c

a b c
11. Prove, using properties of determinants: a  b b  c c  a  a3  b3  c3  3abc .
bc c a a b

. Page # 8
DETERMINANTS

a2 ab ac
12. Prove, using properties of determinants: ab b2 bc  4a2b2c2 .
ac bc c2

13. Prove, using properties of determinants:

a  b2  c2  a 2  2b3 2c 3
3
2a 3 b  c2  a 2  b2  2c 3  abc  a 2  b2  c2 
.
2a 3 2b3 c  a 2  b2  c2 

bc q r y z a p x
14. Without expanding, prove that c  a r  p x  z  2 b q y .
a b pq x y c r z

b2c2 bc b  c
15. Without expanding, prove that c2a2 ca c  a  0 .
a 2b2 ab a  b
9! 10! 11!
16. Using properties of determinants, evaluate: 10! 11! 12! .
11! 12! 13!

1 a 1 1
17. Prove, using properties of determinants: 1 1  b 1  ab  bc  ca  abc .
1 1 1 c

a ab a bc
18. Prove, using properties of determinants: 2a 3a  2b 4a  3b  2c  a3 .
3a 6a  3b 10a  6b  3c

a2 1 ab ac
19. Prove, using properties of determinants: ab b2  1 bc  1  a2  b2  c2 .
ac bc c2 1
2
 b  c a2 a2
2 3
20. Prove that b2 c  a  b2  2abc  a  b  c  .
2
c2 c2  a  b

Multiplication Of Two Determinants :

a1 b1 l m1 a l b l a1 m1  b1 m2
(i )  1  1 1 1 2
a 2 b2 l2 m2 a 2 l1  b2 l2 a 2 m1  b2 m2

(ii) Similarly two determinants of order three are multiplied.


We can write
a1 b1 c1 1 1 1 a11  b11  c11 a12  b12  c1 2 a13  b13  c13
a2 b2 c2 2 2  2 = a 21  b21  c2 1 a 22  b22  c2 2 a 2 3  b23  c2  3
a3 b3 c3 3 3 3 a 31  b21  c3 1 a 3 2  b32  c3  2 a 3 3  b33  c3 3
Here we have multiplied rows by rows. We can also multiply rows by columns or
columns by rows, or columns by columns.
. Page # 9
DETERMINANTS

(iii) If D = |aij| is a determinant of order n, then the value of the determinant |Aij|,
where Aij is the cofactor of aij, is Dn-1. This is known as power cofactor formula.
For a 3 × 3 determinant

a1 b1 c1 A1 B1 C1
If D = 2 b2 c2  0
a then , D² = 2 B2 C2
A where Ai , B i , C i are cofactors
a 3 b3 c 3 A 3 B3 C3

of a i, bi, ci respectively.

a1 b1 c1 A1 A 2 A 3 D 0 0
Proof : Consider a 2 b2 c2 × B1 B2 B3 = 0 D 0
a 3 b3 c 3 C1 C2 C3 0 0 D

Note : a 1 A2 + b1 B 2 + c1 C 2 = 0 etc.

A1 A 2 A 3 A1 A 2 A 3 A1 B1 C1
therefore, B
D. 1 B2 B 3 3
=D  B1 B2 B3 = D² OR
A 2 B2 C2 = D²
C1 C2 C3 C1 C2 C3 A 3 B3 C3

Ex.12 Prove the following by multiplication of determinants and power co-factor formula.
2
0 c b b2  c 2 ab ac a 2 ab ac
c 0 a  ab c2  a2 bc = ab b2 bc  4a 2b2c2
b a 0 ac bc a  b2
2
ac bc c 2

Sol. (i) First determinant is equal to ( 2abc)2


(ii) Second determinant is direct multiplication of determinants in row to row.
(iii) Third determinant is co-factors of the first determinant and therefore square
of the first.
System Of Linear Equation (In Two Variables) :
(i ) Consistent Equations : Definite & unique solution. [intersecting lines]
(ii) Inconsistent Equation : No solution. [Parallel line]
(iii) Dependent equation : Infinite solutions. [Identical lines]
Let a 1 x + b1 y + c 1 = 0 & a 2 x + b2 y + c 2 = 0 then :

a1 b1 c1
  Given equations are inconsistent
a 2 b2 c2 

&

a1 b c
 1  1  Given equations are coincident.
a 2 b2 c2

Applications of Determinant
(i) Area of Triangle : Area of triangle with vertices (x 1 , y1 ), (x 2 , y2 ), (x 3 , y3 ) becomes

x1 y1 1
1 1
| x2 y2 1 | = |x1(y2  y3 )  x2 (y3  y1 )  x 3 (y1  y2 )|
2 x y3 1 2
3

. Page # 10
DETERMINANTS

(ii) Condition of Collinearity : Condition of collinearity of three points (x 1 , y 1 ),

x1 y1 1
(x 2 , y2 ), (x 3 , y3 ) is x2 y2 1 = 0
x3 y3 1

(iii) ax² + 2 hxy + by² + 2 gx + 2 fy + c = 0 represents a pair of straight lines

a h g
if : abc + 2 fgh - af² - bg² - ch² = 0 = h b f
g f c

x y 1
(iv) Equation of a straight line passsing through (x 1 , y 1) & (x 2 , y2) is x1 y1 1 = 0
x2 y2 1

(v) Solution of System of Linear equations (Cramer’s rule)


[Simultaneous Equations Involving Three Unknowns]
Let a 1 x + b1 y + c1 z = d1 ...(I) ; a 2 x + b2 y + c2 z = d2 ...(II) ; a 3 x + b3 y + c3 z = d3 ...(III)
D1 D2 D3
Then, x = , y = , z =
D D D

a1 b1 c1 d1 b1 c1
Where D = a 2 b2 c2 ; D1 = d2 b2 c2 ;
a 3 b3 c3 d3 b3 c3
a1 d1 c1 a1 b1 d1
D2 = a 2 d2 c 2 & D 3 = a 2 b2 d 2
a3 d3 c 3 a 3 b3 d3

Non Homogeneous System :


(a ) If D  0 and alteast one of D 1 , D2 , D3  0 , then the given system of
equations are consistent and have unique non trivial solution .
(b) If D  0 & D1 = D2 = D3 = 0 , then the given system of equations are
consistent and have trivial solution only.
(c) If D = D1 = D2 = D3 = 0 , then the given system of equations are consistent
and have infinite solutions.

a1x  b1y  c1z  d1


In case a 2 x  b2 y  c 2 z  d 2 represents three parallel planes then also
a 3 x  b3 y  c 3 z  d 3

D = D1 = D2 = D3 = 0 but the system is inconsistent.


(d) If D = 0 but atleast one of D1 , D2 , D3 is not zero then the equations are
inconsistent and have no solution .
Homogeneous System :
If x, y, z are not all zero, the condition for a 1 x + b1 y + c1 z = 0; a 2 x + b2 y + c2 z = 0

a1 b1 c1
& a 3 x + b3 y + c3 z = 0 to be consistent in x , y , z is that a 2 b2 c2 = 0.
a 3 b3 c 3

Page # 11
DETERMINANTS
Remember that if a given system of linear equations have Only Zero
Solution for all its variables then the given equations are said to have
Trivial solution.
Ex.13 If the points (a 1 , b1 ), (a 2 , b2 ) and (a 1 + a 2 , b1 + b2 ) are collinear, show that a 1 b2 =
a 2 .b1 .
Sol. The given points are collinear

a1 b1 1
a2 b2 1 0

a1  a 2 b1  b2 1

a1 b1 1
a 2  a1 b2  b1 0  0 [Applying R 2  R2 – R1 , R 3  R3 – R1 ]

a2 b2 0

a1 b1
 0 [Expanding along C 3 ]
a 2  a1 b2  b1

a1 b1
 0 [Applying R 2  R 2 + R 1 ]
a2 b2

 a 1 b2 – a 2 b1 = 0
 a 1 b2 = a 2 b1
Ex.14 Find the equation of the line joining A(1, 3) and B(0, 0) using determinants and
find k if D(k, 0) is a point such that area of ABD is 3 sq. units.
Sol. Let P(x, y) be any position line AB. Then,
Area of ABP = 0

1 3 1
1
 0 0 10
2x y 1

1
 {1(0 – y) – 3(0 – x) + 1 (0 – 0)} = 0
2
 3x – y = 0
This is the required equation of AB.
Now, Area of ABD = 3 sq. units

1 3 1 1 3 1
1
 0 0 1  3  0 0 1  6
2k 0 1
k 0 1

 1(0 – 0) – 3(0 – k) + 1(0 – 0) = 6


 3k = 6
 k = 2

Page # 12
DETERMINANTS
Ex.15 Using cramer’s rule, solve the following system of linear equations.
(a + b)x – (a – b)y = 4ab
(a – b)x + (a + b)y = 2(a 2 – b2 )
Sol. We have,

a  b (a  b)
D  (a  b)2  (a  b)2  2(a 2  b2 )  0
a b ab

So, the given system of equations has a unique solution. Now,

4ab (a  b)
D1  2 2
2(a  b ) (a  b)

2ab (a  b)
 D1  2(a  b) [Taking 2 common from C 1 and (a + b) from R 2 ]
ab 1

 D1 = 2(a + b){2 ab + (a – b) 2 }
 D1 = 2(a + b)(a 2 + b2 )

ab 4ab
and, D2 
a  b 2(a 2  b2 )

a  b 2ab
 D2  2(a  b) [Taking (a – b) common from R 2 and 2 from C 2 ]
1 (a  b)
 D2 = 2(a – b){(a + b)2 – 2ab} = 2(a – b)(a 2 + b2 )
By Cramer’s rule, we have

D1 2(a  b)(a 2  b2 )
x  ab
D 2(a 2  b2 )

D2 2(a  b)(a 2  b2 )
and, y  a b
D 2(a 2  b2 )

Hence, x = a + b, y = a – b is the solution of the given system of equations.

Ex.16 Solve the following system of equations by Cramer’s rule :

2 3 10 4 6 5 6 9 20
   4,    1 and   2
x y z x y z x y z

1 1 1
Sol. Let = u,  v and  w . Then, the above system of equations can be written
x y z
as
2u + 3v + 10w = 4
4u – 6v + 5w = 1
6u + 9v – 20w = 2

2 3 10
Here, D  4 6 5  2(120  45)  3(80  30)  10(36  36)
6 9 20

Page # 13
DETERMINANTS
 D = 150 + 330 + 720 = 1200

4 3 10
D1  1 6 5  4(120  45)  3(20  10)  10(9  12)
2 9 20
 D1 = 300 + 90 + 210 = 600

2 4 10
D2  4 1 5 = 2(–20 – 10) – 4 (–80 – 30) + 10 (8 – 6)
6 2 20

 D2 = –60 + 440 + 20 = 400

2 3 4
and, D3  4 6 1 = 2 (-12-9) -3 (8-6) + 4 (36+36)
6 9 2

 D3 = -42 - 6 + 288 = 240

D1 600 1 1 1
 u      x  2,
D 1200 2 x 2

D2 400 1 1 1
v      y  3,
D 1200 3 y 3

D3 240 1 1 1
and, w       z 5
D 1200 5 z 5
Hence, x = 2, y = 3 and z = 5.
Ex.17 Solve the following system of equations using Cramer’s rule:
5x -7y + z = 11, 6x - 8y - z = 15 and 3x + 2y - 6z = 7.
Sol. The given system of equations is
5x - 7y + z = 11
6x - 8y - z = 15
3x + 2y - 6z = 7

5 7 1
 D  6 8 1  5(48  2)  7(36  3)  1(12  24)
3 2 6

 250  231  36  55

11 7 1
D1  15 8 1 11(48  2)  7(90  7) 1(30  56)
7 2 6

 550  581  86  55

5 11 1
D2  6 15 1  5(90  7)  11(36  3) 1(42  45)
3 7 6

  415  363  3  55

5 7 11
and D3  6 8 15  5(56  30)  7(42  45) 11(12  24)
3 2 7
Page # 14
DETERMINANTS
  430  21  396   55
So, by Cramer’s rule, we have
D1 55 D 55 D 55
x   1, y  2     1 and z  3    1
D 55 D 55 D 55
Hence, x = 1, y = –1 and z = –1 is the solution of the given system of equations.

DRILL - II
cos  cos  cos  sin   sin 
1. Evaluete  sin  cos  0 .
sin  cos  sin  sin  cos 

x sin  cos 
2. Prove that the determinant  sin   x 1 is independent of  .
cos  1 x

x 1 2
3. Find the value(s) of x for which the matrix A  1 0 3 is singular.
5 1 4
 
4. Using determinant, find area of the triangle whose vertices are (0, 0), (4, 3) and (8, 0).
5. Show that the points (1, 0), (6, 0), (0, 0) are collinear.
6. Using determinants, show that the points (a, b + c), (b, c + a) and (c, a + b) are
collinear.
7. Using determinants, find values of a if area of triangle, having vertices (a, 0), (4, 0)
and (0, 2), is 4 sq. units.

1 sin  1
8. If    sin  1 sin  , then prove that 2    4 , for all  .
1  sin  1

9. Solve given system of equation by using determinant method: 3y + x = xy, y + x = 3xy.


10. If A(x1, y1), B(x2, y2) and C(x3, y3) are vertices of an equilateral triangle whose each
2
x1 y1 2
side is equal to a, then prove that x2 y2 2  3a 4 .
x3 y3 2

11. A triangle has its three sides equal to a, b and c. If the coordinates of its vertices
are A(x1, y1), B(x2, y2) and C(x3, y3), show that

2
x1 y1 2
x2 y2 2  (a  b  c)(b  c  a)(c  a  b)(a  b  c)
x3 y3 2

a b c bc  a 2 ca  b2 ab  c2
12. If  = b c a , then then prove that  2 = ca  b2 ab  c2 bc  a 2
c a b ab  c2 bc  a 2 ca  b2

Page # 15
DETERMINANTS
Differentiation of a determinant

f(x) g(x) h(x)


Let (x)  l(x) m(x) n(x) ,
u(x) v(x) w(x)

f (x) g (x) h(x) f(x)


g(x) h(x) f(x) g(x) h(x)
then  (x)  l(x) m(x) n(x)  l '(x) m(x) n(x)  l(x) m(x) n(x)
u(x) v(x) w(x) u(x) v(x) w(x) u(x) v (x) w (x)

x b b
a x b and D = x b d
Ex.18 If D1 = 2 a x then prove that (D1) = 3D2.
a a x dx

R1
Sol. Since we know that if D = f(x) = R2 , then
R3

d R1
(R1 )
d dx d R1
(D) = R2 + (R 2 ) + R2
dx dx
R3 R3 d
(R3 )
dx

d d d x b b x b b
(x) (b) (b)
d dx dx dx d d d a x b
(a) (x) (b)
 (D 1) = a x b + dx dx dx + d d d
dx a a x (a) (a) (x)
a a x
dx dx dx

1 0 0 x b b x b b
= a x b + 0 1 0 + a x b
a a x a a x 0 0 1

x b x b x b x b
= a x + a x + a x = 3 a x = 3D2.

Integration of a Determinant

f(x) g(x) h(x)


Let (x)  a b c , where a, b, c, l, m and n are constants,
l m n

Page # 16
DETERMINANTS
b b b

 f(x)dx
a
 g(x)dx
a
 h(x)dx
a
b
then  (x)dx 
a
a b c
l m n
Note that if more than one row (column) of  (x) are variable, then in order to find
b

  (x ) dx
a
first we evaluate the determinant  (x) by using the properties of

determinants and then we integrate it.

Limit of a Determinant

f(x) g(x) h(x) lim f(x) lim g(x) lim h(x)


x a x a x a

(x)  l(x) m(x) n(x) , then lim (x)  lim l(x) lim m(x) lim n(x) ,
Let x a x a x a x a
u(x) v(x) w(x) lim u(x) lim v(x) lim w(x)
x a x a x a

provided each of nine limiting values exist finitely.

SOLVED EXAMPLES

p b c p q r
Ex.1 If a  p, b  q, c  r and a q c  0 then find the value of p  a  q  b  r  c .
a b r

p b c
R3  R3  R2
Sol. Here   a  p q  b 0 ,
R2  R2  R1
0 bq r c

p b c
pa q b rc
  p  a  q  b  r  c  1 1 0 C1  C1  C2  C3
0 1 1

p b c b c
 
pa q b r c q b r c
  p  a  q  b  r  c  0 1 0
0 1 1

 p b c 
    p  a  q  b r  c      = 0, from the question
p  a q  b r  c 

Page # 17
DETERMINANTS
p b c
But a  p, b  q, c  r . So p  a  q  b  r  c  0
Ex.2 If a, b, c, are pth, qth and rth terms respectively of a H.P., then Find value
bc ca ab
of the determinant p q r
1 1 1

Sol. Let A and d be the first term and common difference of the corresponding A.P.,
then

1 1 1
= A + (p – 1)d, = A + (q – 1)d, = A + (r – 1)d
a b c

1/ a 1/ b 1/ c A  (p  1)d A  (q  1)d A  (r  1)d


Now det. = abc p q r = abc p q r
1 1 1 1 1 1

A A A p 1 q 1 r 1
= abc p q r + abcd p q r = 0 + 0 = 0
1 1 1 1 1 1

1 log x y log x z
Ex.3 Evaluate log y x 1 log y z (where x, y, z being positive).
log z x log z y 1

log y log z
1
log x log x
log x log z
Sol. 1
log y log y
log x log y
1
log z log z

Multiplying R1, R2, R3 by log x, log y and log z respectively


 D = 0 as all rows become identical.
x x x
C1 C2 C3
y
C1 y
C2 y
C3 xyz
Ex.4 Prove that z z z
= (x – y) (y – z) (z – x)
C1 C2 C3 12

x(x 1) x(x 1)(x  2)


x
2 6
y(y 1) y(y 1)(y  2)
y xyz
Sol. Det. = 2 6 =
z(z 1) z(z 1)(z  2) 12
z
2 6

1 x  1 (x  1) (x  2) 1 x x2
xyz 1 y y2
= 1 y  1 (y  1) (y  2) =
1 z  1 (z  1) (z  2) 12 1 z z2

Page # 18
DETERMINANTS

xyz
(by C 2 + C1, C3 + C 1 + 3C 2) = (x – y) (y – z) (z – x)
12

(a  x)2 (a  y)2 (a  z)2


2 2 2
Ex.5 Express D = (b  x) (b  y) (b  z) as the product of two determinants and
(c  x)2 (c  y)2 (c  z)2
evaluate it.

(a 2  2ax  x 2 ) (a 2  2ay  y 2 ) (a 2  2az  z 2 )


2 2 2 2 2 2
Sol. D = (b  2bx  x ) (b  2by  y ) (b  2bz  z )
2 2 2 2 2 2
(c  2cx  x ) (c  2cy  y ) (c  2cz  z )

a2 2a 1 1 x x2
2
y2
= b2 2b 1 1 y
c 2c 1 1 z z2

= 2 ( a – b) ( b – c) ( c – a)( x – y ) ( y – z)( z – x)

x2  x x 1 x  2
2
2x  3x 1 3x 3x  3
Ex.6 If 2
= Px – 12 then find P.
x  2x  3 2x 1 2x 1

Sol. Applying - R 2  R 2 – (R 1 + R 3)

x2  x x 1 x  2
4 0 0
Determinant
x 2  2x  3 2x 1 2x 1

1 2 1 2
Applying = R 1  R1 + x R2 and R3  R3 + x R2
4 4

x x 1 x  2
Determinant = 4 0 0 Applying = R 3  R 3 – 2R 1
2x  3 2x 1 2x 1

x  0 x 1 x  2 x x x 0 1 2
Determinant = 4 0 0 = 4 0 0 + 4 0 0
3 3 3 3 3 3 3 3 3

1 1 1 0 1 2
= x 4 0 0 +
 4 0 0 = (24x – 12)  P = 24.
3 3 3 3 3 3

Ex.7 If f (x) = ax2 + bx + c is a quadratic function such that f (1) = 8, f (2) = 11 and f (-3) =6,
find f (x) by using determinants. Also, find f (0).
Sol. We have, f(x) = ax2 + bx + c
 f (1)  8  a  b  c  8

Page # 19
DETERMINANTS
f (2) 11  4a  2b  c 11
and f ( 3)  6  9a  3b  c  6
This, we obtain the following system of equations
a  bc 8
4a  2b  c 11
9a  3b  c  6

1 1 1
Here, D  4 2 1  1(2  3)  1(4  9)  1(12  18)  5  5  30   20
9 3 1

8 1 1
D1  11 2 1  8(2  3)  1(11  6)  1(33  12)  40  5  45  10
6 3 1

1 8 1
D2  4 11 1  1(11  6)  8(4  9)  1(24  99)  5  40  75   30
9 6 1

1 1 8
and, D3  4 2 11  1(12  33)  1(24  99)  8(12  18)  45  75  240  120
9 3 6

D1 10 1 D 30 3 D 120


 a   , b 2   and c  3  6
D 20 2 D 20 2 D 20

1 2 3
Hence, f (x)  x  x  6
2 2
 f (0) = 6.

a b c c b
Ex.8 Show that a  c b c  a  (a  b  c)(a 2  b2  c2 )
a b ba c

a bc cb
Sol. Let   a  c b c  a Multiplying first column by a, we get
a b ba c

a2 bc cb
1 2
 a  ac b c a
a 2
a  ab b  a c

a 2  b2  c 2 b c c  b
1 2
   a  b2  c 2 b ca [Applying C1  C1  bC2  cC3 ]
a 2
a  b2  c 2 ba c

1 b c c  b
1 2 2 2
   (a  b  c ) 1 b c a [Taking a2 + b2 + c2 common from C1]
a 1 ba c

Page # 20
DETERMINANTS

1 b c c  b
1 2
  (a  b2  c2 ) 0 c a  b Applying R 2  R 2  R1,R 3  R 3  R1
a 0 ac b

1 2 c a b
   (a  b2  c 2 ) 1  Expanding along C1
a ac b

1 2
  (a  b2  c2 ) (bc  a 2  ac  ba  bc)  (a 2  b2  c2 )(a  b  c)
a

1  a 2  b2 2ab 2b
2 2
2ab 1 a b 2a  (1  a 2  b2 )3
Ex.9 2 2
2b  2a 1 a  b

1  a 2  b2 2ab 2b
2 2
Sol. Let   2ab 1  a  b 2a
2b  2a 1  a 2  b2

We shall try to introduce zeros at as many places as possible keeping in mind that
we have to introduce the factor 1  a 2  b2
Applying C1  C1  bC3 and C2  C2  aC3 , we get
1  a 2  b2 0 2b
 0 1  a 2  b2 2a
b(1  a 2  b2 )  a (1  a 2  b2 ) 1  a 2  b2

1 0 2b
   (1  a 2  b2 )2 0 1 2a Taking (1 + a2 + b2) common from both C1 and C2
b a 1  a 2  b 2

1 0 2b
  (1  a 2  b2 )2 0 1 2a Applying R 3  R 3  bR1  aR 2

0 0 1  a 2  b2

1 2a
   (1 a 2  b2 )2 1. Expanding along C1
0 1 a 2  b2

   (1 a2  b2 )3
Ex.10 If A, B and C are the angles of a triangle and

1 1 1
1  sin A 1  sinB 1  sinC 0
,
2 2 2
sinA  sin A sinB  sin B sinC  sin C

Prove that the  ABC is isosceles.

Page # 21
DETERMINANTS

1 1 1
Sol. Let   1  sin A 1  sinB 1  sinC
sinA  sin2 A sinB  sin2 B sinC  sin2 C

Applying C2  C2  C1 and C3  C3  C1 , we get

1 0 0
 1  sin A sinB  sin A sinC  sin A
sin A  sin2 A (sinB  sin A)(sinB  sin A  1) (sinC  sin A)(sinC  sin A  1)

Expanding along R1

sinB  sinA sinC  sinA



(sinB  sinA)(sinB  sinA  1) (sinC  sinA)(sinC  sinA  1)

1 1
= (sinB – sinA) (sinC – sinA)
sinB  sinA  1 sinC  sinA  1

Now   0  (sinB – sinA) (sinC – sinA) (sinC – sinB) = 0

 sinB = sinA or sinC = sinA or sinC = sinB

 B = A or C = A or C = B
In all the three cases, we will have an isosceles triangle.

SINGLE CORRECT TYPE QUESTIONS

sec x sin x tan x


1. 0 1 0 is equal to -
tan x cot x sec x

(A) 0 (B) –1 (C) 1 (D) None

1 0 0
1 3 x3 1
2. The value of x  y is -
5 y3 1

(A) x + y (B) x2 – xy + y 2 (C) x2 + xy + y 2 (D) x3 – y 3


3. If A = (a ij) is a 4 × 4 matrix and c ij is the co-factor of the element a ij in Det
(A), then the expression a11C11+ a12C 12+ a13C 13 + a14C14 equals-
(A) 0 (B) –1 (C) 1 (D) Det. (A)

x 1 2
4. If cofactor of 2x in the determinant 1 2x x  1 is zero, then x equals to
x 1 x 0

(A) 0 (B) 2 (C) 1 (D) –1

Page # 22
DETERMINANTS

a 0 0 p2a 0 0
b c a , then pb c a
5. If  = is equal to -
c a b pc a b

(A) p (B) p2  (C) p3  (D) 2p 

1/a 1 bc
6. The value of the determinant 1/ b 1 ca is equal to
1/ c 1 ab

(A) abc (B) 1/abc (C) 0 (D) None


7. If each row of a determinant of third order of value  is multiplied by 3, then
the value of new determinant is -
(A)  (B) 27  (C) 21  (D) 54 

1 1
2 2
1 2 4
8. The sum of infinite series + 2 + + ........ is -
6 4 2 4 2
4
3

(A) –10 (B) 0 (C) 10 (D) 

a ma  nx x
9. The value of b mb  ny y is -
c mc  nz z

(A) a + b + c (B) x + y + z
(C) m(a + b + c) + n(x + y + z) (D) 0

ka k 2  a 2 1
10. The value of the determinant kb k 2  b2 1 is -
kc k 2  c2 1

(A) k (a + b) (b + c) (c + a) (B) k abc (a2 + b2 + c2)


(C) k (a – b) (b – c) ( c – a) (D) k (a + b – c) (b + c – a) (c + a – b)

r x n(n 1)/2 n

11. If Dr = 2r 1 y n2 , then D
r 1
r is equal to -
3r  2 z n(3n 1)/ 2

1 1 2
(A) n(n + 1)(2n + 1) (B) n (n + 1)2
6 4
(C) 0 (D) None of these

ax ax ax


12. If ax a x ax = 0, then value of x are-
ax ax ax

(A) 0, a (B) 0, – a (C) a, – a (D) 0, 3a

Page # 23
DETERMINANTS

a b c
13. The value of the determinant a 2 b2 c 2 is -
bc ca ab

(A) abc (a – b) (b – c) (c – a)
(B) (a – b) (b – c) (c – a) (a + b + c)
(C) (a – b) (b – c) (c – a) (ab + bc + ca)
(D) None of these

a2 b2 c2 a2 b2 c2
14. If (a 1)2 (b 1)2 (c 1)2 = k a b c , then k is equal to –
(a 1)2 (b 1)2 (c 1)2 1 1 1

(A) 1 (B) 2 (C) 4 (D) 0

x 1 x  2 x  
15. If x is real number such that x2 x 3 x  = 0 then , ,  are in
x 3 x 4 x  

(A) A.P. (B) G.P. (C) H.P. (D) None

1 1 1
16. The determinant a b c is equal to -
2 2 2
a  bc b  ca c  ab

(A) 0 (B) 1 (C) –1 (D) None

1 1 1
m m 1 m2
17. C1 C1 C1 =
m m 1 m2
C2 C2 C2

(A) m(m + 1) (B) m(m – 1) (C) 1 (D) 0

x  2 x  3 x  2a
18. If a, b, c are in A.P., then the value of x  3 x  4 x  2b equals -
x  4 x  5 x  2c

(A) 1 (B) 0 (C) 2a (D) 2x

1  i 1 i i
19. 1 i i 1 i (where i = 1 ) equals -
i 1 i 1 i

(A) 7 + 4i (B) 7 – 4i (C) 4 + 7i (D) 4 – 7i

a ab a bc
20.  = 3a 4a  3b 5a  4b  3c where a = i,b =  , c =  2, then  is equal
6a 9a  6b 11a  9b  6c
to
(A) i (B) – 2 (C)  (D) – i

Page # 24
DETERMINANTS

(x  2)2 (x 1)2 x2
2 2
21. The value of the determinant (x 1) x (x  1)2 is -
x2 (x 1)2 (x  2)2

(A) 0 (B) 8x2 (C) 8 (D) –8

sin(A  B  C) sinB cos C


22. If A + B + C =  , then  sinB 0 tan A equals –
cos (A  B)  tan A 0

(A) 0 (B) 2sin B tan A cosC


(C) 1 (D) None of these
23. The existence of unique solution of the system x + y + z = b, 2x + 3y – z =
6, 5x – y + az = 10 depends on ––
(A) b only (B) a only
(C) a and b (D) neither a nor b
24. Given the system of equations px + y + z = 1, x + py + z = p, x + y + pz = p2,
then for what value of p does this system have no solution –
(A) –2 (B) 1, –2 (C) 1 (D) 0, 1
25. The value of k for which the set of equations 3x + ky – 2z = 0, x + ky + 3z = 0
and 2x + 3y – 4z = 0 has a non – trivial solution is-
(A) 15 (B) 16
(C) 31/2 (D) 33/2

a 2  b2
c c
c
b2  c 2
26. a a is equal to –
a
c2  a 2
b b
b

(A) abc (B) 2abc (C) 4abc (D) 0

23 33 3.22  3.2  1
27. 33 43 3.32  3.3 1 is equal to-
43 53 3.42  3.4 1

(A) 0 (B) 1 (C) 92 (D) None

a b bc ca a b c
28. If bc ca a b =  b c a then  is equal to-
ca a b bc c a b

(A) 1 (B) 2 (C) 3 (D) 4

a b c y b q
29. If  1 = x y z and  2 = x a p then  1 is equal to –
p q r z c r

(A) 2 2 (B) 2 (C) – 2 (D) None


Page # 25
DETERMINANTS

p 15 8
30. If Dp = p2 35 9 , then D + D + D + D + D is equal to –
1 2 3 4 5
p3 25 10

(A) 0 (B) 25 (C) 625 (D) None


31. In a third order determinant each element of the first column consists of sum
of two terms, each element of the second column consists of sum of three
terms and each element of third column consists of sum of four terms, then
it can be decomposed into n determinants, where n has the value –
(A) 1 (B) 9 (C) 16 (D) 24

sin2 A cot A 1
32. For any  ABC, the value of determinant sin2 B cot B 1 is –
sin2 C cot C 1

(A) 0 (B) 1
(C) sin A sin B sin C (D) sin A + sin B + sin C
33. If x, y, z are not all simultaneously equal to zero, satisfying the system of
equations
(sin 3  ) x - y + z = 0
(cos 2  ) x + 4 y + 3 z = 0
2x + 7y + 7z = 0
then the number of principal values of  is
(A) 2 (B) 4 (C) 5 (D) 6

b2c 2 bc b  c
34. If a, b, c are non-zero real numbers, then c2a 2 ca c  a is equal to
a 2b2 ab a  b

(A) abc (B) a2b2c2 (C) ab + bc + ca (D) None

35. If A, B and C are n × n matrices and det(A) = 2, det(B) = 3 and det(C) = 5,


then the value of the det(A2BC –1) is equal to
6 12 18 24
(A) (B) (C) (D)
5 5 5 5

cos (  )  sin(  ) cos 2


36. The determinant sin  cos  sin  is –
 cos  sin  cos 

(A) 0 (B) independent of 

(C) independent of  (D) independent of both  and 

x  1 (x  1)2 x3
x 1 x2 (x  1)3
37. If D(x) = then the coefficient of x in D(x) is
x (x  1) (x  1)3
2

(A) 5 (B) –2 (C) 6 (D) 0

Page # 26
DETERMINANTS
38. If [a] denotes the greatest integer less than or equal to a and – 1  x < 0,
[x] 1 [y] [z]
0  y < 1, 1  z < 2, then [x] [y] 1 [z] is equal to –
[x] [y] [z] 1
(A) [x] (B) [y] (C) [z] (D) None

5 5
C0 C3 14
39. The value of the determinant 5
C1 5
C4 1 is –
5 5
C2 C5 1

(A) 0 (B) –(6!) (C) 80 (D) None

x y z a b c
40. If ax + by + cz = 1, bx + cy + az = 0 = cx + ay + bz, then z x y c a b
y z x b c a
is equal to -
(A) 0 (B) 1 (C) –1 (D) 2

xn x n2 x n3
n
1 1 1
41. If y y n2 y n3 = (x–y) (y–z) (z–x)     , then n is equals to-
z n
z n2 z n3 x y z

(A) 1 (B) –1 (C) 2 (D) –2

  
42. If , ,  a re the roots of x 3 + ax 2 + b = 0, the n the value of    is
  
equals to-
(A) –a 3 (B) a3 –3b (C) a3 (D) a2 – 3b
cos   sin  1
43. The value of the determinant sin  cos  1 is-
cos(  )  sin(  ) 1
(A) independent of  (B) independent of 

(C) independent of  and  (D) none of these

a 2r 216  1 16
44. Let Dr  b 3(4r ) 2(416  1) , then the value of
r 16

Dr is equals to-
c 7(8 ) 4(8  1) r 1
(A) 0 (B) a + b + c
(C) ab + bc + ca (D) none of these
45. The values  and  for which the system of equations x + y + z = 6, x + 2y
+ 3z = 10 and x + 2y +  z =  have unique solution are
(A)   3,   R (B)   3,  =10
(C)   3,  =10 (D)   3,   10
46. The system of linear equations x + y + z = 2, 2x + y – z = 3, 3x + 2y + kz = 4
has a unique solution if-
(A) k  0 (B) –1 < k > 1 (C) –2 < k < 2 (D) k = 0

Page # 27
DETERMINANTS
47. The system of equations :
2x cos2  + y sin2  – 2sin  = 0
x sin2  + 2y sin2  = – 2 cos 
x sin  – y cos  = 0 , for all values of  , can
(A) have a unique non - trivial solution (B) not have a solution
(C) have infinite solutions (D) have a trivial solution

x  3 2x 2  18 3x 3  81
3
48. If f(x) = x  5 2x 2  50 4x  500 then f(1).f(3) + f(3).f(5) + f(5).f(1) is equal to-
1 2 3

(A) f(1) (B) f(3) (C) f(1) + f(3) (D) f(1) + f(5)

1 a b
49. In a  ABC, if 1 c a  0 then sin2 A + sin2 B + sin2 C is equal to-
1 b c

9 4
(A) (B) (C) 1 (D) 3 3
4 9

1 1 1
50. cos(nx) cos(n  1)x cos(n  2)x is independent of
sin(nx) sin(n  1)x sin(n  2)x

(A) x (B) n
(C) Both x and n (D) None of these

51. If x, y, z are integers in A .P , lying between 1 and 9 , and x51, y 41 and z31 are

5 4 3
three digited numbers, then value of x51 y41 z31 is
x y z

(A) xyz (B) xyz


(C) 0 (D) None of these

mx mx  p mx  p
52. If f '(x) = n n  p np then y = f(x) represents
mx  2n mx  2n  p mx  2n  p

(A) a straight line parallel to x- axis


(B) a straight line parallel to y- axis
(C) parabola
(D) a straight line with negative slope

6 2i 3 6
53. If   12 3  8 i 3 2  6 i ,then  is
18 2  12 i 27  2 i

(A) an integer (B) a natural number


(C) an irrational number (D) an imaginary number
Page # 28
DETERMINANTS

sin x cos x cos x


54. The number of distinct real roots of cos x sin x cos x  0 in the interval
cos x cos x sin x


0 x  is
4
(A) 0 (B) 2 (C) 1 (D) 3

6x  5y  z  0
55. The system of equations 3x  y  4z  0 has
x  2y  3z  0

(A) only a trivial solution for   R

(B) exactly one nontrivial solution for some real 

(C) infinite number of nontrivial solutions for one value of 


(D) none of these.

MULTIPLE CORRECT TYPE QUESTIONS

a a  d a  2d
1. Let   a  d a  2d a , then
a  2d a a d

(A)  depends on a (B)  depends on d


(C)  is a constant (D) All of these

sin 3 2 3
2. The value of  satisfying cos 2 8 7  0 is
2 14 11

  n 
(A) n (B) n  (C) 2n  (D) n   1
6 6 6
3. The system of equations

 a  b  x  ay  bz  0
 b  c  x  by  cz  0 has a non-trival solution , if
 a  b  y   b  c  z  0
(A) a,b,c are in A.P
(B) a,b,c are in G.P
(C) a,b,c are in H.P

(D)  is a root of ax 2  2bx  c  0

Page # 29
DETERMINANTS

n n 1 n2
n n1 n2
4. Let f  n  Pn Pn1 Pn 2 ,
n n1 n2
Cn Cn1 Cn2

where the symbols have their usual meanings. The f(n) is divisible by

(A) n2  n 1 (B)  n  1! (C) n! (D) None

5. The values of  and  for which the system of equations.

x  y  z  6, x  2y  3z  10, x  2y  z   have no solution are

(A) 3 (B)   10 (C) 3 (D)   10

Tp Tq Tr
th th th
6. If Tp ,Tq ,Tr are the p ,q and r terms of an A.P ., then p q r cannot be
1 1 1

equal to
(A) 1 (B) -1 (C) 0 (D) p+q+r

2r x n  n  1 n

7. 2
If Sr  6r  1 y n2  2n  3  , then the value S
r 1
r is independent of
4r3  2nr z n3  n  1

(A) x (B) y (C) n (D) z

e2iA eiC eiB


iC
8. If A  B  C  , ei  cos   i sin  and z  e e2iB eiA then
eiB eiA e2iC

(A) Re(z) = 4 (B) Im(z) = 0 (C) Re(z) = –4 (D) Im(z) = –1

1
f  x f    f x 
1 x
9. If f(x) is a polynomial satisfying f  x   and f (2) = 17 , then f (5)
2 1
1 f 
x

is not equal to
(A) 126 (B) 626 (C) 124 (D) 624

a a2 0
2
10. Let   1 2a  b  a  b  then
0 1 2a  3b

(A) a  b is factor of D (B)  a  2b  is factor of D

(C) a2 is a factor of D (D) (2a + b) is factor of D

Page # 30
DETERMINANTS
11. The set of equations x – y + 3z = 2, 2x – y + z = 4, x – 2y + az = 3 has
(A) unique soluton only for a = 0
(B) unique solution for a  8
(C) infinite number of solutions for a = 8
(D) no solution for a = 8

a2 a 2  (b  c)2 bc
12. The determinant b2 b2  (c  a)2 ca is divisible by :
c2 c 2  (a  b)2 ab

(A) a + b + c (B) (a + b) (b + c) (c + a)
(C) a2 + b2 + c2 (D) (a - b) (b - c) (c - a)
13. The set of equations x – y + 3z = 2 , 2x – y + z = 4 , x – 2y + az = 3 has
(A) unique soluton only for a = 0 (B) unique solution for a  8
(C) infinite number of solutions for a = 8 (D) no solution for a = 8

1 a a2
1 x x2
14. If = 0 , then :
b2 ab a 2

1 a
(A) x = a (B) x = b (C) x = (D) x =
a b
15. Which of the following determinant(s) vanish(es)?

1 ab 1
a 
1
1 b c b c (b  c) b
1 c a c a (c  a) 1 bc 1  1
c
(A) (B) b
1 a b a b (a  b) 1
1 ca c  1
a

0 a b a c log x x y z log x y log x z


ba 0 bc log y x y z 1 log y z
(C) (D)
ca cb 0 log z x y z log z y 1

a1  b1x a1x  b1 c1
16. If ai,bi,ci  R(i = 1,2,3)and x  R and a 2  b2 x a 2 x  b2 c 2  0 , then
a 3  b3 x a 3 x  b3 c3

(A) x = 1 (B) x = -1

a1 b1 c1
a b2 c2  0
(C) 2 (D) x = 0
a3 b3 c3

1 1 1
2
17. Let f1(x) = x + a, f2(x) = x + bx + c and   f1 (x1 ) f1 (x 2 ) f1 (x 3 ) , then
f 2 (x1 ) f 2 (x 2 ) f 2 (x 3 )

(A)  is independent of a (B)  is independent of b and c


(C)  is independent of x1, x2, x3 (D)  depends on a
Page # 31
DETERMINANTS

COMPREHENSION TYPE QUESTIONS


Passage - 1

x a a
If a, b  0 and   x   b x a , then
b b x

1.   x  is increasing in

(A)  ab, ab  (B)  ,   


ab  ab,  
(C)  3
ab, 3 ab  (D)  ab,ab
2.   x  is decreasing in

(A)  ab, ab  (B)  3


ab, 3 ab 
(C)  ,  3
 
ab  3
ab,   (D)  ab,ab 
3.   x  has a local minimum, at

(A) x  3 ab (B) x   3 ab

(C) x  ab (D) x   ab
Passage - 2
Let   0 and  c denotes the determinant of cofactors, then c  n1 , where
n (> 0) is the order of  .
On the basic of above information, answer the following questions:
4. If a, b, c are the roots of the equation x3 – px2 + r = 0, then the value of
bc  a 2 ca  b2 ab  c2
ca  b2 ab  c2 bc  a 2
is
ab  c2 bc  a 2 ca  b2
(A) p2 (B) p4 (C) p6 (D) p9
5. If I 1, m1, n1; I2, m2, n2; I3, m3, n3 are real quantities satisfying the six relation:

I12  m12  n12  I22  m22  n22  I23  m23  n23  1

I2I3  m2m3  n2n3  I3I1  m3m1  n3n1

I1 m1 n1
I m2 n2
= I1I2  m1m2  n1n2  0 , then the value of 2 is
I3 m3 n3

(A) 0 (B) 1 (C) 2 (D) 3

Page # 32
DETERMINANTS
6. If a, b, c are the roots of the equation x3 – 3x2 + 3x + 7 = 0, then the value of

2bc  a 2 c2 b2
c2 2ac  b2 a2
is
b2 a2 2ab  c2

(A) 9 (B) 27 (C) 81 (D) 0

MATRIX MATCH TYPE QUESTIONS

1 x x x2
x 1  x x 2  ax 5  bx 4  cx 3  dx 2  ex  f
1. Let , then
x2 x 1 x

Column - I Column - II
(A) The value of ‘f’ is equal to (p) 0
(B) The value of ‘e’ is equal to (q) 1
(C) The value of ‘a + c’ is equal to (r) –1
(D) The value of ‘b + d’ is equal to (s) 3

1  sin2 x cos2 x sin2x


2. ,  are the ,maximum and minimum values of f(x)  sin2 x 1  cos2 x sin2x ,
sin2 x cos2 x 1  sin2x

then
Column - I Column - II

(A)  87 (p) 6

(B) 2  311 (q) 2


(C) f   (r) 4
2


(D) f  (s) –2
2

INTEGER TYPE QUESTIONS


1. The number of positive integral solutions of the equation

x 3  1 x2 y x 2z
xy 2 y 3  1 y 2 z = 11 is
xz 2 yz 2 z 3  1

Page # 33
DETERMINANTS

1 x x2 x3 1 0 x  x4
x x2 1 3 0 x  x4 x3  1
2. If , find the value of .
x2 1 x x  x4 x3 1 0

3. 2x + 3y – 3z = 0
5x – 2y + 2z = 19
x + 7y – 5z = 5, Find the value of x  y  z
4. For what value of 2k/33 the equations
x + ky + 3z = 0, 3x + ky – 2z = 0, 2x + 3y – 4z = 0
possess a nontrival solution over the set of rationals ?

1 a bc 1 a a2
5. The value of 1 b ca  1 b b2 is -
1 c ab 1 c c2

a2 (s  a)2 (s  a)2
6. If 2s  a  b  c and (s  b)2 b2 (s  b)2  ks3 (s  a)(s  b)(s  c)
(s  c)2 (s  c)2 c2

then the numerical quantity k should be


7. If a,b,c  R , the determinant

a2  b2  c2 bc  ca  ab bc  ca  ab
bc  ca  ab a2  b2  c2 bc  ca  ab is always greater than or equal to k, find k
bc  ca  ab bc  ca  ab a2  b2  c2

1  a 2  b2 2ab  2b
8. If a,b  R then   2ab 1  a  b2 2a
2
 k 3a 2 b2 , find k.
2b  2a 1  a 2  b2

1  sin2 x cos2 x 4sin 2x


2 2
9. Find the maximum value of f  x   sin x 1  cos x 4sin 2x
sin2 x cos2 x 1  4sin 2x

p  sin x q  sin x p  r  sin x


10. If p, q, r, s are in A.P. and f (x) = q  sin x r  sin x 1  sin x such that
r  sin x s  sin x s  q  sin x
2

 f(x)dx = – 4 then common difference of the A.P is  k , find k


0

Page # 34
DETERMINANTS

PREVIOUS YEARS QUESTIONS (AIEEE)

log  p 1
1. If p , q , r term of a GP are  , m, n then the value of
th th th log m q 1 is equal to-
log n r 1
[AIEEE 2002]
(A) 0 (B) 1 (C)  +m+n (D) None
2. If a1, a2, a3,......, an,..... are in G.P., then the value of the determinant

log a n log a n 1 log a n  2


log a n  3 log a n  4 log a n  5 , is- [AIEEE 2004,05]
log a n  6 log a n  7 log a n  8

(A) 0 (B) 1 (C) 2 (D) –2


3. The system equations  x + y + z =  – 1, x +  y + z =  – 1, x + y +  z =  – 1 has
no solution, if  is - [AIEEE 2005]
(A) –2 (B) either –2 or 1
(C) not – 2 (D) 1

1  a 2 x (1  b2 )x (1  c2 )x
4. If a2 + b2 + c2 = – 2 and f(x) = (1  a 2 )x 1  b2 x (1  c2 )x then f(x) is a polynomial of
(1  a 2 )x (1  b2 )x 1  c2 x

degree [AIEEE 2005]


(A) 1 (B) 0 (C) 3 (D) 2

1 1 1
5. If D = 1 1  x 1 for x  0, y  0 then D is- [AIEEE 2007]
1 1 1 y

(A) divisible by neither x nor y


(B) divisible by both x and y
(C) divisible by x but not y
(D) divisible by y but not x
6. Let a, b, c be any real numbers. Suppose that there are real numbers x, y, z not all
zero such that x = cy + bz, y = az + cx, and z = bx + ay. Then a2 + b2 + c2 + 2abc is
equal to [AIEEE 2008]
(A) –1 (B) 0 (C) 1 (D) 2
7. Let a, b, c be such that b(a + c)  0.

a a 1 a 1 a 1 b 1 c 1
If b b  1 b  1 + a  1 b 1 c  1 =0,
c c 1 c 1 (1)n  2 a (1)n 1b (1)n c

then the value of n is : [AIEEE 2009]


(A) any even integer (B) any odd integer
(C) any integer (D) zero
Page # 35
DETERMINANTS
8. Consider the system of linear equations: [AIEEE 2010]
x1 + 2x2 + x3 = 3
2x1 + 3x2 + x3 = 3
3x1 + 5x2 + 2x3 = 1.
The system has
(A) exactly 3 solutions
(B) a unique solution
(C) no solution
(D) infinite number of solutions
9. The number of values of k for which the linear equations 4x + ky + 2z = 0;
kx + 4y + z = 0; 2x + 2y + z = 0 possess a non-zero solution is [AIEEE 2011]
(A) 2 (B) 1 (C) zero (D) 3
10. The number of values of k, for which the system of equations:
(k + 1)x + 8y = 4k
kx + (k + 3)y = 3k – 1 [JEE Mains 2013]
has no solution is:
(A) 2 (B) 3 (C) infinite (D) 1
3 1  f(1) 1  f(2)
2 2 2
11. If ,  0 and f(n)     and 1  f(1) 1  f(2) 1  f(3)  k(1   ) (1  ) (  ) , then k is
n n

1  f(2) 1  f(3) 1  f(4)

equal to [JEE Mains 2014]


(A) 1 (B) –1 (C)  (D) 1 / 
12. The set of all values of  for which the system of linear equations :
2x1 – 2x2 + x3 =  x1
2x1 – 3x2 + 2x3 =  x2
– x1 + 2x2 =  x3
has a non-trivial solution [JEE MAINS 2015]
(A) Is an empty set (B) Is a singleton
(C) Contains 2 elements (D) Contains more than 2 elements
13. The system of linear equations [JEE MAINS 2016]
x  y  z  0
x  y  z  0
x  y  z  0
has a non-trivial solution for :
(A) exactly one value of  . (B) exactly two values of  .
(C) exactly three values of  . (D) infinitely many values of  .

Page # 36
DETERMINANTS

PREVIOUS YEARS QUESTIONS (IIT JEE)

Single Correct Queations:


1. The parameter , on which the value of the determinant

1 a a2
cos  p  d  x cos px cos  p  d  x does- not depend on: [IIT-1997]
sin  p  d  x sin px sin  p  d  x

(A) a (B) p (C) d (D) x

xp  y x y
2. The determinant yp  z y z  0 , if [IIT-1997]
0 xp  y yp  z
(A) x, y, z are in A.P. (B) x, y, z are in G.P
(C) x, y, z are in H.P (D) xy, yz, zx are in A.P.

1 x x 1
2x x  x  1  x  1 x
3. If f(x) = then f(100) is equal to [IIT-1999]
3x  x  1  x  x  1 x  2   x  1 x  x  1

(A) 0 (B) 1 (C) 100 (D) –100


4. If the system of equations x  ky  z  0, kx  y  z  0, x  y  z  0 has non zero solution,
then possible values of k are [IIT-2000]
(A) –1, 2 (B) 1, 2 (C) 0, 1 (D) –1, 1

sin x cos x cos x


5. The number of distinct real roots of cos x sin x cos x = 0 in the interval
cos x cos x sin x

 
– < x < is [IIT 2001]
4 4
(A) 0 (B) 2 (C) 1 (D) 3
6. The number of values of K for which the system of equations, (K + 1) x + 8y = 4K
and Kx + (K + 3) y = 3K – 1 has infinitely many solutions, is [IIT 2002]
(A) 0 (B) 1 (C) 2 (D) Infinite
1 1 1
1 3
7. Let  = – + i . Then the value of the determinant D = 1  1   2 is
2
2 2 1 2 4
[IIT 2002]
2
(A) 3  (B) 3  (  – 1) (C) 3  (D) 3  (1 –  )
8. If x + ay = 0 ; y + az = 0 ; z + ax = 0, then value of ‘a’ for which system of equations
will have infinite number of solutions is [IIT 2003]
(A) a = 1 (B) a = 0
(C) a = –1 (D) no value of a

Page # 37
DETERMINANTS
9. If the system of equations 2x – y – 2z = 2 ; x – 2y + z = – 4 ; x + y +  z =
4 has no solutions the  is equal to [IIT 2004]

(A) –2 (B) 3 (C) 0 (D) –3


10. Consider the system of equations [IIT 2008]
x – 2y + 3z = –1
–x + y – 2z = k
x – 3y + 4z = 1
STATEMENT-1 : The system of equations has no solution for k  3
1 3 1
STATEMENT-2 : The determinant 1 2 k  0, for k  3
1 4 1
(A) Statement–1 is True, Statement–2 is True; Statement–2 is a correct
explanation for Statement–1.
(B) Statement–1 is True, Statement–2 is True; Statement–2 is not a correct
explanation for Statement–1
(C) Statement–1 is True, Statement–2 is False
(D) Statement–1 is False, Statement–2 is True
11. Let P = [aij] be a 3 × 3 matrix and let Q = [bij], where bij = 2i + jaij for 1  i, j  3. If the
determinant of P is 2, then the determinant of the matrix Q is [IIT 2012]
(A) 210 (B) 211 (C) 212 (D) 213
12. Which of the following values of  satisfy the equation

(1   )2 (1  2)2 (1  3)2
(2   )2 (2  2 )2 (2  3 )2  648 ? [JEE Adv. 2015]
(3   )2 (3  2 )2 (3  3 )2

(A) –4 (B) 9 (C) –9 (D) 4


Subjective Questions
1. Let the three digit numbers A28, 3B9, where A, B and C are integers between

A 3 6
0 and 9 , be divisible by a fixed integer K. Show that the determinant 8 9 C is
2 B 2

divisible by K. [IIT-1990]

p b c
p q r
2. If a  p,b  q,c  r and a q c  0 Then find the value of  
pa q b r c
a b r

[IIT-1991]
3. For a fixed positive integer n, if

n!  n  1 !  n  2  !
 D 
D  n  1 !  n  2 !  n  3  ! then show that  3
 4  is divisible by n [IIT-1992]
  n! 
 n  2 !  n  3  !  n  4  !

Page # 38
DETERMINANTS

4. Let  and  be real. Find the set of all values of  for which the system of linear
equations [IIT-1993]

x   sin   y   cos   z  0 , x   cos   y   sin   z  0

 x   sin   y   cos   z  0 has a non-trivial solution for   1, find all values of  .


5. For all values of A,B,C and P,Q,R ; show that [IIT-1994]
cos  A  P  cos  A  Q  cos  A  R 
cos  B  P  cos  B  Q  cos  B  R   0
cos  C  P  cos  C  Q  cos  C  R 
6. Let a  0,d  0 Find the value of the determinant : [IIT-1996]

1 1 1
a a a  d   a  d  a  2d 
1 1 1
a  d a  d  a  2d   a  2d  a  3d 
1 1 1
 a  2d   a  2d  a  3d   a  3d  a  4d 
bc ca ab
p q r
7. Find the value of the determinant where a,b and c are respectively
1 1 1

the pth, qth and rth terms of a harmonic progression. [IIT-1997]

8. Suppose f  x  is a function satisfying the following conditions : [IIT-1998]

(A) f(0) = 2, f(1) = 1


(B) f has a minimum value at x = 5/2 and

2ax 2ax  1 2ax  b  1


(C) for all x, f '  x   b b 1 1
2  ax  b  2ax  2b  1 2ax  b

where a,b are some constants. Determine the constants a , b and the function f (x)

sin  cos  sin2


 2   2   4 
9. Prove that for all values of  sin     cos     sin  2   = 0 [IIT-2000]
 3   3   3 
 2   2   4 
sin     cos     sin  2  
 3   3   3 

10. Let a, b, c be real numbers with a2 + b2 + c2 = 1. Show that the equation

ax  by  c bx  ay cx  a
bx  ay  ax  by  c cy  b 0 represents a straight line [IIT-2001]
cx  a cy  b  ax  by  c

Page # 39
DETERMINANTS

ANSWER KEY
DRILL - I
1. (-202) 16. 2(9!) (10!) (11!)

DRILL - II
1. (1) 3. (-3) 4. 12 sq. units 7. a = 0, 8 9. x = -1, y =1/4
12. x = 2, y = 3, z = 5
SINGLE CORRECT TYPE QUESTIONS
1. C 2. C 3. D 4. C 5. B 6 C
7. B 8. A 9. D 10. C 11. C 12. D
13. C 14. C 15. A 16. A 17. C 18. B
19. C 20. A 21. D 22. A 23. B 24. A
25. D 26. C 27. A 28. B 29. B 30. D
31. D 32. A 33. C 34. D 35. B 36. B
37. A 38. C 39. D 40. B 41. B 42. C
43. A 44. A 45. A 46. A 47. B 48. B
49. A 50. B 51. C 52. A 53. A 54. C
55. C
MULTIPLE CORRECT TYPE QUESTIONS
1. AB 2. ACD 3. BD 4. AC 5. AD 6. ABD
7. ABCD 8. BC 9. ACD 10. AB 11. BD 12. ACD
13. BD 14. AD 15. ABCD 16. ABC 17. AB

COMPREHENSION TYPE QUESTIONS


1. B 2. A 3. C 4. C 5. B 6. D

MATRIX-MATCH TYPE QUESTIONS


1. A-q, B-s, C-r, D-q. 2. A-r, B-p, C-s, D-q
INTEGER TYPE QUESTIONS
1. (3) 2. (9) 3. (1) 4 (1) 5. (0) 6. (2)
7. (0) 8. (3) 9. (6) 10. (1)
PREVIOUS YEAR QUESTIONS (AIEEE)
1. A 2. A 3. A 4. D 5. B 6. C
7. B 8. C 9. A 10. D 11. A 12. C
13. C

Page # 40
DETERMINANTS
PREVIOUS YEAR QUESTIONS (IIT JEE)
1. B 2. B 3. A 4. D 5. A 6. B
7. B 8. C 9. D 10. A 11. D 12. BC

Subjective question
2. (2)

n  
4.     2 , 2  ,    (1)n . 
2 8 8

4d 2
6.
a(a d)2 (a 2d)3 (a  3d)2 (a  4d)
7. (0)

1 2 5 1 5
8. f(x)  x  x  2 , a  ,b  
4 4 4 4

Page # 41
MATRICES 2
UNIT

 Theory

 Drill Exercises
CONTENTS
 Solved Examples

 Exercises

 Single Correct Type Questions

 Multiple Correct Type Questions

 Comprehension Type Questions

 Matrix Match Type Questions.

 Integer Type Questions

 Previous Years Questions

 Answer Key
MATRICES
Definition :
A rectangular arrangement of numbers in rows and columns, is called a Matrix.
This arrangement is enclosed by small ( ) or big [ ] brackets. A matrix is
represented by capital letters A, B, C etc. and its element are by small letters
a, b, c, x, y etc.

3 4  a b c   1 3 c 
eg. A = 5 6  , B = d e f  , C =  2 9 4  , D = [1, 4, 9],
     
Order of Matrix :
A matrix which has m rows and n columns is called a matrix of order m × n
(m by n).
A matrix A of order m × n is usually written in the following manner-

 a11 a12 a13 ...a1j ...a1n 


 a 21 a 23 a 23 ...a 2 j ...a 2n 
 
 ..... ..... ..... ..... ..... 
A =  or
a i1 a i2 a i3 ...a ij ...a in 
 
 ..... ..... ..... ..... ..... 
 a m1 a m2 a m3 ...a mj ...a mn 

i  1, 2,......m
A = a ij  m × n where j  1, 2,......n

Here a ij denotes the element of ith row and jth column.


Ex.1 A matrix A = [a ij] of order 2 × 3 whose elements are such that a ij = i + j
then find the matrix A.
Sol. aij is the element of ith row and jth column of matrix A
 a11 = 1 + 1 = 2, a 12 = 1 + 2 = 3, a 13 = 1 + 3 = 4
a21 = 2 + 1 = 3, a 22 = 2 + 2 = 4, a 23 = 2 + 3 = 5

a11 a12 a13  2 3 4


A = a a a  = 3 4 5 
 21 22 23   

Type of Matrices
1. Row matrix :
If in a Matrix, there is only one row, then it is called a Row Matrix.

Thus A = a ij  m x n is a row matrix if m = 1.


2. Column Matrix :
If in a Matrix, there is only one column, then it is called a Column Matrix.
Thus A = [a ij]m × n is a Column Matrix if n = 1.
3. Square Matrix :
If number of rows and number of column in a Matrix are equal, then it is called
a Square Matrix.

Thus A = a ij  m x n is a Square Matrix if m = n

Page # 43
MATRICES

 a11 a12 a13 


a 21 a 22 a 23 
eg.   is a Square Matrix of order 3 × 3.
a 31 a 32 a 33 

Note :
(a) If m  n then Matrix is called a Rectangular Matrix.
(b) The elements of a Square Matrix A for which i = j i.e. a 11, a 22 , a 33 , ....
ann are called diagonal elements and the line joining these elements is
called the principal diagonal or leading diagonal of Matrix A.
(c) Trace of a Matrix : The sum of diagonal elements of a square matrix A
is called the trace of Matrix A which is denoted by tr A.
n

trA = a
i 1
ii = a 11 + a 22 + ... a nn

4. Singleton Matrix :
If in a Matrix there is only one element then it is called Singleton Matrix.
Thus

A = a ij  m x n is a Singleton Matrix if m = n = 1.

5. Null or Zero Matrix :


If in a Matrix all the elements are zero then it is called a zero Matrix and it
is generally denoted by O.

Thus A = a ij  m x n is a zero matrix if a ij = 0 for all i and j.

6. Diagonal Matrix :
If all elements except the principal diagonal in a Square Matrix are zero, it is
called a Diagonal Matrix.
Thus a Square Matrix

A = a ij  is a Diagonal Matrix if a ij = 0, when i  j

2 0 0 
 
eg. 0 3 0  is a diagonal Matrix of order 3 × 3, which also can be denoted by
 0 0 4 
diag (2, 3, 4)
Note :
(a) No element of Principal Diagonal in diagonal Matrix is zero.
(b) Number of zero in a diagonal matrix is given by n 2 – n where n is a order
of the Matrix.
7. Scalar Matrix :
If all the elements of the diagonal of a diagonal matrix are equal, it is called
a scalar matrix. Thus a Square Matrix A = a ij  is a Scalar Matrix if

aij = 0
k
i j
i  j where k is a constant.

Page # 44
MATRICES
8. Unit Matrix :
If all elements of principal diagonal in a Diagonal Matrix are 1, then it is called
Unit Matrix. A unit Matrix of order n is denoted by I n.
Thus a square Matrix

A = a ij  is a unit Matrix if

1 0 0 
aij = 
1 i j
0 i j eg. I3
 
= 0 1 0 
0 0 1 
9. Triangular Matrix :

A Square Matrix a ij  is said to be triangular matrix if each element above or


below the principal diagonal is zero.It is of two types-
(a) Upper Triangular Matrix : A Square Matrix [aij] is called the upper triangular
Matrix, if aij = 0 when i > j.

3 1 2
 
eg. 0 4 3 is a upper triangular matrix of order 3 × 3
 0 0 6 

(b) Lower Triangular Matrix : A Square Matrix a ij  is called the lower
Triangular Matrix, if
aij = 0 when i < j

1 0 0 
 
eg. 2 3 0  is a lower triangular matrix of order 3 × 3.
 4 5 2 

n  n  1
Note : Minimum number of zero in a triangular matrix is given by
2
where n is order of Matrix.
10. Equal Matrix :
Two Matrix A and B are said to be equal Matrix if they are of same order and
their corresponding elements are equal

1 6 3  a1 a 2 a 3 
eg. if A = 5 2 1  and B = b b b  are equal Matrix then
   1 2 3
a1 = 1, a 2 = 6, a 3 = 3, b1 = 5, b2 = 2, b3 =1
11. Singular Matrix :
Matrix A is said to be singular matrix if its determinant |A| = 0, otherwise non-
singular matrix i.e.
If det |A| = 0  Singular
and det |A|  0  non - singular

1 3 2
 5  is a singular matrix, then find k.
Ex.2 If A = 2 k
4 2 1 

Sol. A is singular  |A| = 0

Page # 45
MATRICES

1 3 2

2 k 5 = 0
4 2 1

 1(k – 10) + 3(2 – 20) + 2(4 – 4k) = 0


 7 k + 56 = 0  k = – 8

Algebra of Matrices
1. Addition of Matrices :

If A = a ij  m x n and B = bij  m x n are two matrices of the same order then their
sum A + B is a matrix whose each element is the sum of corresponding
elements.

5 2  1 5 
   2 2
eg. If A = 1 3 and B = 3 3 
4 1   

 5  1 2  5 6 7 
1  2 3  2  
then A + B =   = 3 5 
4  3 1  3  7 4 
Properties of Matrices addition :
If A, B and C are Matrices of same order, then-
(a) A + B = B + A ( Commutative Law)
(b) (A+ B) + C = A + (B + C) ( Associative Law)
(c) A + O = O + A = A, where O is zero matrix which is additive identity
of the matrix.
(d) A + ( – A) = 0 = (–A ) + A where (–A) is obtained by changing the sign
of every element of A which is additive inverse of the Matrix
(e ) tr (A ± B) = tr (A) ± tr (B)

 3 2 1 2
Ex.3 If X and Y two matrices are such that X – Y =  1 0  and X + Y = 3 4 
   
then find Y.

 3 2
Sol. Given that X – Y =  1 0  ...(1)
 

1 2
and X + Y = 3 4  ...(2)
 
Subtracting (2) from (1)

 3 2 1 2
– 2Y =  1 0  – 3 4 
   

 3  1 2  (2) 2 4
(–2)Y =  1  3 0  4  =  4 4
   

1 2 4  1 2
 Y = –  4 4 =  2 2 
2

Page # 46
MATRICES
Ex.4 If  is an imaginary cube root of unity, show that

1  2    2 1  2 1 
     
 2 1   2 1   1  2 
 2      is a null matrix.
 1    1  2    2 1 

1  2    2 1  2 1 
 2
    
  1   2 1   1  2 
Sol.  2     
 1    1  2    2 1 

1    2 1    2 1    2  0 0 0
 
 1    2 1    2 1    2   0 0 0
 , which is a null matrix.
2 2 2  0 0 0
1     1     1       

2. Scalar Multiplication of Matrix :

Let A = a ij  m x n be a matrix and k be a number then the matrix which is


obtained by multiplying every element of A by k is called scalar multiplication
of A by k and it is denoted by kA.

2 4 10 20 
 1  5
eg. If A = 3 then 5 A = 15
4 6 20 30 

Properties of Scalar Multiplication :


If A, B are Matrices of the same order, then
(a) k(A + B) = kA + kB
(b) tr (kA) = k tr (A)

1 a  2 3 1 3 
Ex.5 If X = 0 1  and 3X – 0 2 = 0 1  then find a.
     

 3 3a 
Sol. 3X =  0 3 
 

 3  2 3a  3 1 3a  3
 L.H.S. = 0  0 3  2  = 0 1 
  
Now by equality of two matrices, we have 3a – 3 = 3  a = 2
3. Multiplication of Matrices :
If A and B be any two matrices, then their product AB will be defined only when
number of column in A is eq ual to the number of rows in B . If

A = a ij  m x n and B = [bij ]n x p then their product AB = C = cij  , will be matrix


n

of order m x p, where (AB)ij = C ij = a


r 1
ir brj

Page # 47
MATRICES

1 2 
1 4 2  
Ex.6 If A = 2 3 1 and B = 2 2 ,then find AB.
   1 3 

1. 1  4. 2  2. 1 1. 2  4. 2  2. 3 
Sol : AB =  2. 1  3. 2  1. 1 2. 2  3. 2  1. 3 
 

11 16 
AB =  9 13 
 
Properties of Matrix Multiplication :
If A, B and C are three matrices such that their product is defined , then
(a) AB  BA (Generally not commutative)
(b) (AB) C = A (BC) (Associative Law)
(c) IA = A = AI, I is identity matrix for matrix multiplication
(d) A(B + C) = AB + AC (Distributive Law)
(e ) If AB = 0 It does not mean that A = 0 or B = 0, again product of two non-
zero matrix may be zero matrix.
(f) tr(AB) = tr(BA)
(g) The multiplication of two diagonal matrices is again a diagonal matrix.
(h) The multiplication of two triangular matrices is again a triangular matrix.
(i) The multiplication of two scalar matrices is also a scalar matrix.
(j) The positive integral powers of a matrix A are defined only when A is a
square matrix. Also then
A2 = A.A A3 = A.A.A = A2A

2 3 1 x
 2 1
Ex.7 If [1 x 2] 0 4  1 = 0, then find the value of x.
0 3 2  
Sol. The LHS of the equation

x
 
= [2 4x + 9 2x + 5]  1 
 1
= [2x + 4x + 9 – 2x – 5] = 4x + 4
Thus 4x + 4 = 0  x = – 1

 2 1
Ex.8 If A =  1 2  and A2 – 4A – nI = 0, then n is equal to -
 

 5 4   8 4  n 0 
Sol. A2 =  4 5  , 4A =  4 8  , nI =  0 n 
     
 A2 – 4A – nI

 5  8  n 4  4  0 
=  4  4  0 5  8  n 
 

 3  n 0 
=  0  3  n 

Page # 48
MATRICES
 A2 – 4A – nI = 0

 3  n 0  0 0 
  0 3  n  = 0 0

 – 3 – n = 0
 n = – 3

 cos  sin  
Ex.9 If E(  ) =   sin  cos   , then find the value of E (  ). E(  ).
 

Sol. E(  ) . E(  )

 cos  sin    cos  sin  


=   sin  cos     sin  cos 

 cos  cos   sin  sin  cos  sin   sin  cos  


=   sin  cos   cos  sin   sin  sin   cos  cos  

 cos(  ) sin(  ) 
=   sin(   ) cos(  ) = E(    )

 0  tan  /2 cos   sin  


Ex.10 If matrix P =  tan  /2 0  then prove that (I – P)  sin  cos   = I + P.
  

1 0   0  tan ( / 2)
Sol. I – P = 0 1  –  tan ( /2) 0 
  

 1 tan ( / 2)
=   tan ( / 2) 1 

cos   sin  
 ( I – P)  sin  cos  
 

 1 tan   /2  cos   sin  


=   tan   /2 1  .  sin  cos  
   

 cos   tan   /2 sin   sin   tan   /2 cos 


=   tan   / 2 cos   sin  tan   /2 sin   cos  
 

1  2sin2 ( /2)  2sin2 ( /2)


=   tan( /2)(2cos2 ( /2)  1)  2sin( / 2)cos( /2)

2sin( / 2)cos( / 2)  tan( /2)(2cos2 ( /2)  1 



tan( /2)(2sin( /2)cos( /2))  (1  2sin2 ( /2))

 1  tan   / 2 
= tan   / 2 1  = I + P
 

Page # 49
MATRICES
Tanspose of a Matrix :
The matrix obtained from a given matrix A by changing its rows into columns or
columns into rows is called transpose of Matrix A and is denoted by AT or A´.
From the definition it is obvious that
If order of A is m × n, then order of A T is n × m.

 a1 b1 
a a a3  a
eg. Transpose of Matrix  b1 b2 b2 
 1 2 b3  2 x 3 is  2 
a 3 b3  3 x 2

Properties of Transpose :

eA j
T
T
(a) = A

(b) If A and B are the matrices of same order then (A  B)T = AT  B T


(c) (AB)T = BT AT
(d) (kA)T = k(A)T
(e ) (A1A2A3 ......An–1An) T = AnT An–1T.......A3TA2TA1T
(f) Transpose of an identity matrix is the matrix itself.
(g) Trace of a matrix and its transpose remains same.

1 2  3 4 
Ex.11 If A = 3 0  and B = 1 6  then find (AB)T.
   

 3  2 4  12 5 16
Sol. AB = 9  0 12  0  = 9 12
   

5 9
 (AB)T = 16 12 
 

Symmetric and Skew-Symmetric Matrices


Symmetric Matrix :
A square matrix A = [a ij] is called symmetric matrix if a ij = a ji for all i,j or
AT = A

a h g  a h g 
   
eg. A = h b f  , then AT = h b f 
 g f c   g f c 

n  n  1
Note : Maximum number of different element in a symmetric matrix is .
2
Skew - Symmetric Matrix :
A square matrix A = [a ij] is called skew - symmetric matrix if
a ij = – a ij for all i, j

0 h
g

f
or AT = – A eg.  h 0
0 
 g f
Note : All Principal diagonal elements of a skew - symmetric matrix are always
zero.

Page # 50
MATRICES
Properties of Symmetric and skew- symmetric matrices :
1. If A is a square matrix, then A + A T, AAT, ATA are symmetric matrices while
A – AT is Skew- Symmetric matrices.
2. If A is a Symmetric Matrix, then –A , KA, A T, An , A–1 , B TAB are also
symmetric matrices where n  N , K  R and B is a square matrix of order
that of A.
3. If A is a skew symmetric matrix, then-
(a) A2n is a symmetric matrix for n  N
(b) A2n+1 is a skew - symmetric matrices for n  N.
(c) kA is also skew - symmetric matrix where k  R.
(d) B TAB is also skew – symmetric matrix where B is a square matrix of
order that of A
4. If A, B are two symmetric matrices, then-
(a) A  B, AB + BA are also symmetric matrices.
(b) AB – BA is a skew - symmetric matrix.
(c) AB a symmetric matrix when AB = BA.
5. If A is a skew - symmetric matrix and C is a column matrix, then C T AC
is a zero matrix.
6. Every square matrix A can uniquelly be expressed as sum of a symmetric
1 T  1 T 
 
and skew symmetric matrix i.e. A =  A  A  +  A  A 
2  2 
 
Ex.12 Express A as the sum of a symmetric and a skew symmetric matrix, where
4 2 3
 6 .
A= 1 3
 5 0 7

 4 2 3  4 1 5 
Sol. We have A =  1 3 6 and A   2 3 0
 5 0 7  
  3 6 7 

 4 2 3  4 1 5   8 3 8 
Then A  A   1 3 6 +  2 3 0  =  3 6 6  ................(i)
 5 0 7  3 6 7  8 6 14
   

4 2 3  4 1 5  0 1 2
and A  A    1 3 6 –  2 3 0  =  1 0 6 ................(ii)
 5  
 0 7  3 6 7 
   2 6 0 
Adding (1) and (ii) we get

8 3 8   0 1 2 

2A =  3 6 6    1 0 6
 8 6 14  2 6 0 

 4 3 / 2 4  0 1/2 1 
 A= 3 /2 3  
3  1/2 0 3

 4 3 7   1 3 0 

Symmetric matrix Skew symmetric matrix

Page # 51
MATRICES
DRILL - I

 x  3 z  4 2y  7   0 6 3y  2
 6 a  1 0   6 3 2c  2  , then find x, y, z, a, b, c.
1. If   = 2b  4 21
 b  3  21 0   0 

2. What is the number of all possible matrices of order 3  3 with each entry 0 or 1?
3. If A= diag [-2, 5, 1] and B=diag [1, 0, -4],then find 3A - 5B.

 1 3 2 2 1 1
4. If A   2 0 2 and B   , then find the matrix C such that 5A + 3B + 2C
   1 0 1
is a null matrix.

1 2 2
5. If A  2 1 2 , then prove that A2 - 4A - 5I =O.
 2 2 1
 

0 1 0 
6. If A  0 0 1  , then prove that A3 = aI + bA + cA2.
a b c 
 

2 3 1
7. If A   1 0 2  and f  x   x 2  5x  7 , then find f(A).
 1 1 1
 

0 0
8. If A    , then find (i) A20, (ii) A35.
 2 0 

1 4 0 3
9. If A    and B T   , then find (7A + 5B)T.
 1 3  1 2

 2 1 3   1 1 
10. If A   4 1 0  and B   0 2 . then show that (AB)T = BT AT.
  5 0
 

5 2 a 
11. If A  b c 3  is a symmetric matrix, find a, b, c and d.
4 d 7 
 

3 4 
12. Express A   as the sum of symmetric and a skew symmetric matrix.
 1 1

0 2b c 
13. If A  a b c  , then find the values of a, b, c such that AT A = I.
a  b c 
 
14. If A and B are symmetric matrices, then prove that
(i) AB - BA is a skew symmetric matrix.
(ii) AB + BA is a symmetric matrix.
15. Show that the matrix B T AB is symmetric or skew symmetric according as A is
symmetric or skew symmetric matrix.

Page # 52
MATRICES
16. For a square matrix A, prove that
(i) A + AT is symmetric matrix.
(ii) A - AT is skew symmetric matrix.
(iii) A AT is symmetric matrix.
(iv) AT A is symmetric matrix.

3 4 
17. If A   , then prove that A n  1  2n 4n  , for n  N using principle of math
1 1  n 1  2n 
ematical induction.
n n n n
18. If A=diag [a, b, c], then prove that A  diag a ,b ,c  , for all n  N .

 cos x sin x  n  cos n x sin n x 


19. If A x    , then prove that  A x    , for all n  N .
  sin x cos x    sin n x cos n x 

Determinant of a Matrix :

 a11 a12 a13 


 a 23  be a square matrix, then its determinant, denoted by |A|
If A = a 21 a 22 
a 31 a 32 a 33 

a11 a12 a13


or Det (A) is defined as |A| = a 21 a 22 a 23
a 31 a 32 a 33

Properties of the Determinant of a Matrix :


(i) |AB| = |A||B|
(ii) |AT| = |A|
(iii) |kA| = kn |A|, if A is a square matrix of order n.
(iv) If A is a skew symmetric matrix of odd order then |A| = 0
(v) If A = diag (a 1,a 2.......a n ) , then |A| = a 1a2 ...a n
(vi) |A|n = |An| , n  N.

Adjoint of a Square Matrix :


If every element of a square matrix A be replaced by its cofactor in |A|, then
the transpose of the matrix so obtained is called the adjoint of matrix A and
it is denoted by adj A. Thus if A = [a ij] be a square matrix and C ij be the cofactor
of a ij in |A|, then Adj. A = [C ij]T

 a11 a12 ....a1n  T


a  C11 C12 ... ... C1n 
....a 2 n 
 21 a 22  C21 C22 ... ... C2n 
Hence if A = .... .... ........  , then Adj. A =  ... ... ... ... ... 

.... .... ........  
 ... ... ... ... ... 
a an 2 ....a n n 
 n1  
Cn1 Cn2 ... ... Cnn 

Properties of Adjoint Matrix :


If A, B are non - singular square matrices of order n and I n is corresponding
unit matrix, then
(i) A (adj. A) = |A| I n = (adj A) A. (Thus A (adj A) is always a scalar matrix)

Page # 53
MATRICES
(ii) |adj A| = |A|n–1
(iii) adj (adj A) = |A|n–2 A
2
(iv) |adj ( adj A)| = |A| n 1

(v) adj (AT) = (adj A)T


(vi) adj (AB) = (adj B) (adj A)
(vii) adj (kA) = kn–1 (adj. A), k  R
(viii) A is symmetric  adj A is also symmetric

1 3 5 
 
Ex.13 If A = 3 5 1  , then find adj. A.
 5 1 3 
T
 14 4 22  14 4 22
 4 22 14   4 22 14 
Sol. adj. A =  = 
 22 14 4   22 14 4 

2 0 0 
 
Ex.14 If A = 2 2 0  , then find adj (adj A).
 2 2 2 
2 0 0
Sol. |A| = 2 2 0 = (2) (2) (2) = 8
2 2 2
2 0 0  1 0 0 
   
Now adj (adj A) = |A|3–2 A = 8 2 2 0  = 16 1 1 0 
2 2 2  1 1 1 

1 0 3 
 
Ex.15 If A = 2 1 1  , then find | adj (adj A) | .
0 0 2 

1 0 3 
 
Sol. |A| = 2 1 1  = 2
0 0 2 
2 2
 |adj (adj.A) | = |A| n 1 = | A|2 [  Here n = 3]
= 24 = 16
Inverse of a Square Matrix :
A square matrix A said to be invertible (non singular) if there exists a matrix
B such that, A B = I = B A
B is called the inverse (reciprocal) of A and is denoted by A-1 . Thus
A -1 = B  A B = I = B A .
We have, A . (adj A) = |A| In
A -1 A (adj A) = A -1 In|A|
In (adj A) = A -1|A|In
(adj A)
 A -1 =
|A|
Page # 54
MATRICES
Note :
(i) Matrix A is called invertible if it is non - singular.
(ii) Inverse of a matrix is unique.
Properties of Inverse Matrix :
Let A and B are two invertible matrices of the same order, then
(i) (AT)–1 = (A–1)T
(ii) (AB)–1 = B –1 A–1
(iii) (Ak)–1 = (A–1)k, k  N
(iv) adj (A–1) = (adj A)–1
(v) (A–1)–1 = A
1
(vi) |A–1| = = | A | –1
| A|

(vii) If A = diag (a 1,a 2,....,a n), then A–1 = diag (a 1–1, a 2–1,.....,a n–1 )
(viii) A is symmetric matrix  A–1 is symmetric matrix.
(ix) A is diagonal matrix  A–1 is diagonal matrix.
(x) AB = AC  B = C, iff |A|  0.

0 1 2
 
Ex. 16 Find the inverse of the matrix A  1 2 3
3 1 1

Sol. A  0  11  2  3  3  4   1  3  2  0

Hence matrix A is non-singular.


Co-factors of the 1st, 2nd and 3rd rows of |A| are 1, 8,  5; 1,  6, 3;  1, 2,  1

1 8 5
Therefore the matrix formed by co-factors of |A| is C   1 6 3 
 
1 2 1
 1 1 1
 8 6 2 
 Adj A = Transpose of C =  
 5 3 1

 1 1 1 1/2 1/2 1/2


1 1
 A 1
 Adj.A    8 6 2  or A 1   4 3 1 
A 2  
 5 3 1 5 /2 3 /2 1/2

 1 2 2
Ex.17 Find the inverse of the matrix A   1 3 0  by using elementary row
 0 2 1 
 
transformations.
 1 2 2 1 0 0 
 1 3 0   0 1 0  A
Sol. We have, A = I A or  0 2 1  0 0 1 
   
Page # 55
MATRICES

1 2 2 1 0 0 
 0 5 2  1 1 0  A [Applying R2  R 2 + R 1]
0 2 1  0 0 1 
   

1 2 2 1 0 0 
 0 1 0   1 1 2  A [Applying R 2  R 2 + 2R3]
0 2 1  0 0 1 
   

1 0 2 1 2 4


 0 1 0    1 1 2  A [Applying R 1  R 1 + (–2)R2, R3  R 3 + 2R 2]
0 0 1   2 2 5 
   

1 0 0   3 2 6 
 0 1 0   1 1 2  A [Applying R1  R 1 + 2R3]
0 0 1   2 2 5 
   

3 2 6 
Hence, A–1 = 1 1 2
2 2 5 
 
Matrix Polynomial :
If f (x) = a0xn + a1xn – 1 + a2xn – 2 + ......... + anx0 then we define a matrix polynomial
f (A) = a0An + a1An–1 + a2An–2 + ..... + anIn
where A is the given square matrix. If f (A) is the null matrix then A is called the
zero or root of the polynomial f (x).

 2 0 1
Ex.18 Let A   2 1 3  and f(x) = x2 – 5x + 6. Find f(A)
 1 1 0 
 

Sol : f  A   A 2  5A  6I3 .

2
 2 0 1  2 0 1 1 0 0 
 2 1 3   5  2 1 3   6 0 1 0 
=    1 1 0  0 0 1 
 1 1 0     

 2 0 1   2 0 1  10 0 5   6 0 0 
  2 1 3   2 1 3  10 5 15    0 6 0 
  1  1 0   1 1 0   5 5 0   0 0 6 
       

 2  2  0  2  1 1 2  0  0  1  1 1 2 1  0  3  1  0 
  2  2  1  2  3  1 2  0  1  1  3  1 2 1  1 3  3  0 
 
 1 2   1 2  0  1  1 0   11  0  1  11   1 3  0  0 
 6  10 00 0 5   3 1 2   4 0 5 
 0  10 6  5 0  15  =  3 2 5    10 1 15 
+ 
 0  (5) 0  ( 5) 6  0   4 1 4  5
   5 6 

 3  4 1  0 2  5   1 1 3 
 3  10 2  1 5  15   7 1 10 
4  5 1  5 4  6  1 4 2 
  

Page # 56
MATRICES
Special Matrices :
(a) Orthogonal Matrix : A square matrix is called orthogonal iff AAT = I.
(b) Idempotent Matrix : A square matrix is idempotent provided A2 = A.

 2 2 4
e.g. The matrix A   1 3 4  is idempotent as
 1 2 3
 

 2 2 4  2 2 4   2 2 4
A2  1 3 4   1 3 4   1 3 4 
 1 2 3  1 2 3   1 2 3
    

(c) Nilpotent Matrix: A square matrix is said to be nilpotent matrix of order m, m  N,


if Am = O , A m 1  O . Least possible m for which Am = O is called index of nilpotent
matrix.

1 1 3
5 2 6
Ex .19 Show that the matrix   is a nilpotent matrix of index 3.
 2 1 3 

1 1 3 0 0 0 
Sol. Let A   5 2 6   A   3 3 9 
  2

 2 1 3  1 1 3 

 0 0 0   1 1 3  0 0 0 
A  A .A   3 3 9    5 2 6   0 0 0 
3 2

1 1 3  2 1 3 0 0 0 

 A3 = 0 i.e., A is a nilopotent matrix of index 3.


(d) Periodic Matrix : A square matrix is which satisfies the relation AK+1 = A, for some
positive integer K, is a periodic matrix. The period of the matrix is the least value of
K for which this holds true.
Note that period of an idempotent matrix is 1.
(e) Involutary Matrix : If A2 = I , the matrix is said to be an involutary matrix.
Note that A = A–1 for an involutary matrix.
(f) Conjugate of a Matrix:
The matrix obtained from any given matrix A containing complex number as its
elements, on replacing its elements by the corresponding conjugate complex numbers
is called conjugate of A and is denoted by A .

1  2i 2  3i 3  4i  1  2i 2  3i 3  4i 
e.g. A  4  5i 5  6i 6  7i  . Then A  4  5i 5  6i 6  7i 
 8  8 7 
 7  8i 7   7  8i

(g) Transpose Conjugate of a Matrix:


The transpose of the conjugate of a matrix A is called transposed conjugate of A
and is denoted by A . The conjugate of the transpose of A is the same as the
transpose of the conjugate of A i.e. (A ')  (A)' = A
Page # 57
MATRICES

If A = [aij]m´n, then A = [bji]n´m where bji = a ij


i.e. the (j, i)th element of A = the conjugate of (i, j)th element of A.

1  2i 2  3i 3  4i  1  2i 4  5i 8 
4  5i 5  6i 6  7i  2  3i 5  6i 7  8i 
e.g. If A =  then A = 

 8 7  8i 7  3  4i 6  7i 7 

Properties of Transpose Conjugate:


(i) (A) = A
(ii) (A + B) = A + B

(iii) (kA) = k A, k being any number


(iv) (AB) = BA
(h) Hermitian and Skew-Hermitian Matrix:

A square matrix A = [aij] is said to be Hermitian matrix if aij = a ij  i, j i.e. A = A and


a square matrix, A = [aij] is said to be a skew-Hermitian if aij = – a ij ,  i, j i.e. A = -A.

 3i 3  2i 1  i 
 0 2  i  3  2i 2i 2  4i  are skew-Hermitian matrices.
e.g. 2  i 0  ,  1  i
  2  4i 0 

 3 3  4i 5  2i 
 a b  ic  
b  ic , 3  4i 5 2  i  are Hermitian matrices
d  
 5  2i 2  i 2 

System of Simultaneous Linear Equations


Consider the following system of n linear equations in n unknowns:
a11 x1 + a12 x2 + ………+ a1n xn = b1
a21 x1 + a22 x2 + ………+ a2n xn = b2
. . . .
. . . .
an1 x1 + an2 x2 + ………+ ann xn = bn
This system of equation can be written in matrix form as

a11 a12 ............a1n   x1   b1 


a 21 a 22 ...........a 2n  x2   b2 
     
. . .  .   . 
. . .  .  .  or AX = B
. . .  .  . 
a a ...........a  x  b 
 n1 n2 nn  nn  n  n1  n  n1

Then matrix A is called the coefficient matrix of the system of linear equations.

Homogeneous and Non-Homogeneous System of Linear Equations:


A system of equations AX = B is called a homogeneous system if B = O, where O is
a null matrix. Otherwise, it is called a non-homogeneous system of equations.

Page # 58
MATRICES
Consistent System:
If the system of equations has one or more solutions, then it is said to be a consistent
otherwise inconsistent.
Solution of a Non-Homogeneous System of Linear Equations:
There are two methods of solving a non-homogeneous system of simultaneous linear
equations.
(i) Cramer’s Rule (ii). Matrix Method
Matrix Method:
Consider the equations
a1x + b1y + c1z = d1
a2x + b2y + c2z = d2
a3x + b3y + c3z = d3 ......… (i)

 a1 b1 c1  x d1 
If A  a 2 b2 c2  , X   y  and D  d 2 
  z   
a 3 b3 c3    d 3 
(i) If A is non singular then the system of equations given by AX = D has a unique
solution given by X = A-1D.
(ii) If A is singular matrix, and (adjA)D = O, then the system of equations given by
AX = D is consistent with infinitely many solutions.
(iii) If A is singular matrix, and (adjA)D  O, then the system of equation given by
AX = D is inconsistent and has no solution.
Solution of Homogeneous System of Linear Equations:
Let AX = O be a homogeneous system of n linear equation with n unknowns. Now if
A is non-singular then the system of equations will have a unique solution i.e.
trivial solution and if A is a singular then the system of equations will have infinitely
many non trivial solutions.
Ex.20 With the help of matrices, solve the equations;
3x + y + 2z = 3, 2x – 3y – z = –3, x + 2y + z = 4.
Sol. We can write the given equations as AX = B …(1)

3 1 2  x  3 
Where, A  2 3 1 , X   y  , B   3
1 2 1  z  4 
     

3 1 2 
Since, |A| 2 3 1 = 3 (-3 + 2 ) –1 (2 + 1) + 2 (4 +3) = -3 –3 + 14 = 8  0
1 2 1 
 

From (1), we have X = A–1B …(2)

 1 3 5   1 3 5 
1 1
Now, adj A   3 1 7  and A  3 1 7 
 7 5 11 8  
   7 5 11

 1 3 5   3  1
1 1
A 1B  3 1 7  .  3    2
8  7 5 11  4 8  1 
     

Page # 59
MATRICES
Hence, from (2)

x   1 
 y    2  x = 1, y = 2, z = –1
 z   1 
   
Matrices of Rotation of Axes
We know that if x and y axis are rotated through an angle  about the origin the
new co-ordinates are given by
x = X cos  –Y sin 
y = X sin  + Y cos 

 x   cos   sin    X  cos   sin  


     , where  is the matrix of rotation through
y
   sin  cos   Y  sin  cos  
an angle 

DRILL - II
3 2 
1. Find inverse of the matrix 7 5  by using Elementry Row Transformations.
 

0 1 2
 
2. Find inverse of 1 2 3 by using Elementry Row Transformation method.
 3 1 1 

2 5 4 3 
3. If A    and B   , then prove that AB  A B .
 2 1   2 5 

1 0 1 
4. If A  0 1 2 , then show that 3A  33 A .
0 0 4 
 

5. If a matrix A of order 3  3 has determinant 2, then find the value of A  8I .

6. If A and B are square matrices of order 3 such that A  1 and B  3 , then find the
value of |7AB|.
7. If A and B are square matrices of the same order such that |A| = 6 and AB = I, then
write the value of |B|.
8. If A is a square matrix such that AT A=I, write the value of |A|.
9. If A is a skew symmetric matrix of order 3, write the value of |A|.

2 1 4 
10. Find the adjoint of the given matrix B  1 2 1  .
0 3 1
 
11. A trust fund has Rs. 30000 that is to be invested in two different types of bonds. The
first bond pay 5% interest per annum which will be given to an orphange and the
second bond pays 7% interest per annum which will be given to ‘Cancer Aid Soci-
ety’. an NGO. Using matrix multiplication, determine how to divide Rs. 30000 among
two types of bonds if the trust fund obtains an annual total interest Rs. 1800. What
are the values reflected in the question?

Page # 60
MATRICES

cos   sin  0 
12. If A   sin  cos  0  , then proe that A(adj A)=|A|I3=(adj A)A.
 0 0 1 

1 0 0 
13. Find the inverse of the following matrix: 3 3 0  .
5 2 1
 

1 2 2
14. If A  2 1 2 , then find A-1 and hence prove that A2 - 4A - 5I =O.
2 2 1
 
15. If A is an invertible matrix of order 3 × 3 and |A| = 5, then find |adj A|.

5 0 0 
16. If A is a matrix of order 3 × 3 such that A  adj A   0 5 0  , then find |adj A|.
0 0 5 
 
17. If A is an invertible matrix of order 3 × 3 and |A| = 4, then find |adj(adj(A)|.
1
   
 1  tan 
2  1 tan 
2 cos   sin   .
18. Show that     
tan

1    tan

1   sin  cos  
 2   2 

19. If A   1 tan   , then prove that T 1 cos 2  sin 2  .


A A 
  tan  1   sin 2 cos 2 
20. Use matrix method to examine the given system of equations for consistency or
inconsistency: 4x + 3y =5, 8x + 6y = 10.
21. Use matrix method to examine the given system of equations for consistency or
inconsistency: x + y + z =3, 2x - y +z = -1, 2x +y - 3z = -9.
22. Use matrix method to examine the given system of equations for consistency or incon-
sistency: 2x - y + 3z = 5, 3x + 2y - z = 7, 4x + 5y - 5z = 9.

2 3 5 
23. If A  3 2 4 , then find A-1, Using A-1, solve the system of equations:
1 1 2
 
2x - 3y + 5z = 11, 3x + 2y - 4z = -5, x + y - 2z = -3.

1 1 0   2 2 4
  
24. Use the product 2 3 4  4 2 4 to solve the system of equations:
0 1 2  2 1 5 
x - y - 3 = 0, 2x + 3y + 4z = 17, y + 2z = 7.
25. The sum of three numbers is 6. If we multiply the third number by 3 and odd second
number to it, we get 11. By adding the first and third numbers, we get double of the
second number. Represent it algebrically and find the numbers using matrix method.
26. The cost of 4 kg onion, 3 kg wheat and 2 kg rice is Rs. 60. The cost of 2kg onion, 4kg
wheat and 6 kg rice is Rs. 90. The cost of 6 kg onion, 2kg wheat and 3 kg rice is Rs.
70. Find cost of each item per kg by matrix method.

Page # 61
MATRICES

SOLVED EXAMPLES
Ex.1 If A is idempotent and A + B = I, then which of the following is true ?
(A) B is idempotent (B) AB = 0
(C) BA = 0 (D) All of these
Sol. (D) Here A + B = I  B = I – A
Now B2 = (I – A) (I – A)
= I2 – AI – IA + A 2
= I – A – A + A2
= I – A – A + A since A 2 = A A is idempotent
= I – A = B
 B is idempotent is true
Again AB = A (I – A) = AI – A 2 = A – A = 0
Also BA = (I – A) A = IA – A 2 = A – A = 0
Hence all statements are true .

 1 2 2 
Ex.2 If  2 1 2  is an orthogonal matrix then k is equal to -
 2 2 1
 
(A) 1 (B) 1/2 (C) 1/3 (D) None of these
Sol. (C) Here let

 1 2 2   1 2 2 
A = k  2 1 2   AT = k  2 1 2 
 2 2 1  2 2 1
   
Since A is orthogonal  AAT = I

 1 2 2   1 2 2 
 2 1 2   2 1 2 
 k2  2 2 1  2 2 1
   

 1 4  4 2  2  4 2  4  2
 422 
= k2 2  2  4 4 1  4
 2  4  2 4  2  2
 4  4  1 

9 0 0 
 9 0
= k2 0 2
0 0 9  = 9k I
 

1 1
 9k2 = 1  k2 =  k = ±
9 3
Ex.3 If A and B are non singular Matrices of same order then Adj. (AB) is
(A) Adj. (A) (Adj. B) (B) (Adj. B) (Adj. A)
(C) Adj. A + Adj. B (D) none of these
Sol. (B) A adj A = |A| I
(AB) (adj AB) = |AB| I
Also (AB)(adj B . adj A) = A (B adj B) adj A
= A |B| In AdjA
= |B| A adj A
= |B| |A| In or |AB| In
Page # 62
MATRICES

x   x x 
 x x   x  , then A–1 exists if
Ex.4 Let A = 
 x x x   

(A) x  0 (B)   0
(C) 3x +   0,   0 (D) x  0,   0
Sol. (C)

x x x 3x   x x 1 x x
x x x = 3x   x x = (3x + ) 1 x  
 x
We have |A| = 
x x x 3x   x x 1 x x

1 x x
= (3x +  ) 0  0 =  2 (3x +  )
0 0 

[Take 3x +  common and use R2  R2 – R1, R3  R3– R1]


Thus, A–1 will exist if   0 and 3x +   0

 cos 2  sin  cos    cos 2  sin  cos  


Ex.5 If A =  sin  cos  sin 2
  ; B =  sin  cos  sin2  
  
are such that, AB is a null matrix, then which of the following should necessarily

be an odd integral multiple of .
2
(A)  (B)  (C)  –  (D)  + 
Sol. (C)

 cos 2  sin  cos    cos 2  sin  cos  


AB =    
 sin  cos  sin 2    sin  cos 
 sin 2
 

 cos 2  cos 2   sin  cos  sin  cos  cos 2  sin  cos   sin  cos  sin 2  
=  
2 2
 cos  sin  cos   sin  sin  cos  sin  cos  sin  cos   sin 2  sin 2  

 cos  cos  cos(  ) cos  sin  cos(  ) 


=  
 sin  cos  cos(  ) sin  sin  cos(  ) 
  –  must be an odd integral multiple of  /2

1 1
Ex.6 The number of solution of the matrix equation X2 =   is
 2 3
(A) more than 2 (B) 2 (C) 1 (D) 0

a b
Sol. (A) Let X =  
c d

Page # 63
MATRICES

 a 2  bc ab  bd 
X2 =  2


 ac  cd bc  d 
a2 + bc = 1
ab + bd = 1  b(a + d) = 1
b 1
ac + cd = 2  c (a + d) = 2  
c 2
bc + d2 = 3  (d2 – a2) = 2  (d – a) (a + d) = 2
d – a = 2b (using bc = 1 – a2)
a + d = 1/b
__________________
2d = 2b + 1/b 2a = 1/b – 2b
d = b + 1/2b a = 1/2b – b
c = 2b

 2 1  2 2 1
 b  2  1  2b  3  3b  2
 4b  4b 2

1 1 1
3x + =2  b=  or b = 
4x 6 2

 0 1/ 2   0 1/ 2   2 / 6 1/ 6 
Matrices are   ;   ;  
 2 2   2  2   2 / 6 4 / 6 
  
Ex.7 Matrix A satisfies A2 = 2A – I where I is the identity matrix then for n  2, An is
equal to (n  N)
(A) nA – I (B) 2n – 1A – (n – 1)I
(C) nA – (n – 1)I (D) 2n – 1A – I
Sol. (C) A2 = 2A – I  A3 = 2A2 – IA = 2(2A – I) – A
A3 = 3A – 2I A4 = 3A2 – 2A = 3(2A – I) – 2A
A4 = 4A – 3I A5 = 5A – 4IM
An = nA – (n – 1)I
Ex.8 Let three matrices

 2 1 3 4   3  4
A =  4 1 ; B = 2 3 and C =  2 3 
  
then evaluate

 ABC   A( BC) 2   A( BC)3 


tr(A) + tr  
 + tr     + ....... +
 2  4  + tr  8   =
   

3 4   3  4  1 0
Sol. BC = 2 3  2 3   BC = 0 1 = I

A A 1 1
tr(A) + tr   + tr  2  + ....... = tr(A) + tr(A) + 2 tr(A) + .......
2 2  2 2

Page # 64
MATRICES

t r (A )
= = 2 tr(A) = 2(2 + 1) = 6 Ans.
1  1 2

 l1 m1 n1 
 n 2  , where < l1, m1, n1 > , < l2 , m2, n2 > and < l3, m3, n3 > are the
Ex.9 If A = l2 m2

 l3 m3 n 3 
direction cosines of three mutually perpendicular straight lines, then prove that
AA = I.
Sol. Since < l1, m1, n1 > , < l2 , m2, n2 > and < l3, m3, n3 > are the direction cosines of
three mutually perpendicular straight lines

l12  m12  n12  1, l22  m22  n 22 1 and l32  n32  m32 1


and l1l2 + m1m2 + n1n2 = l2l3 + m2m3 + n2n3 = l3l1 + m3m1 + n3n1 = 0

 l1 m1 n1   l1 l2 l3 
 n2  
We have A = l2 m2
 A   m1 m2 m3  .
 
 l3 m3 n 3   n1 n2 n 3 

 l1 l2 l3   l1 m1 n1   l12 l1m1 l1n1 


   1 0 0 
A 'A  m1 m 2 m3  l2 m2 
n 2   m1l1 m12 m1n1 
     0 1 0  I
 n1 n 2 n 3  l3 m3 n 3   n1l1 m1n1 n12  0 0 1
 
Hence proved.

0 1
Ex.10 Show that the matrix A =  satisfies A2 = – I. Hence or otherwise find the 16th
1 0 

1 1
power of the matrix  .
1 1 

0 1  0 1  1 0 
Sol. A2     I
1 0  1 0   0 1

1 1  0 1  1 0
Let B = 1  
 1   1 0   0 1  = A + I

B2 = (A + I) (A + I) = A2 + 2A + I
Since A2 = –I, B2 = 2A

1 0   256 0 
B16 = (B2)8 = (2A)8 = 28 (A2)4 = 28 (–I)4 = 28  0 1    0 256 
  

Page # 65
MATRICES

SINGLE CORRECT TYPE QUESTIONS


1. If a matrix B is obtained by multiplying each element of a matrix A of order
2 × 2 by 3, then relation between A and B is -
(A) A = 3B (B) 3A = B (C) 9A = B (D) A = 9B

5 7   1 2 4 5 
x 1   3 5   
2. If   –   = 4 4  then -
2 6   2 y 0 4 

(A) x = 1, y = – 2 (B) x = – 1, y = 2
(C) x = 1, y = 2 (D) x = –1, y =– 2

16 10  0 0 
3. If  6 8  + 2B = 0 0 , then B is equal to -
   

 8 5   8 5   8 5   8 5 
(A)  3 4  (B)  3 4  (C)  3 4 (D) 3 4 
       
4. If A and B are matrices of order m × n and n × n respectively, then which of
the following are defined-
(A) AB, BA (B) AB, A2 (C) A2, B 2 (D) AB, B 2

3 1 
5. If A = 7 5  and A2 + kI = 8A, then k equals -
 
(A) 4 (B) 8 (C) 1/4 (D) 1/16
6. If A,B,C are matrices of order 1 × 3, 3 × 3 and 3 × 1 respectively, the order
of ABC will be
(A) 3 × 3 (B) 1 × 3 (C) 1 × 1 (D) 3 ×1

0 1 1   x 
  
7. The root of the equation [x 1 2] 1 0 1   1 = 0 is -
1 1 0   1 

(A) 1/3 (B) – 1/3 (C) 0 (D) 1

1 2  3 2
8. If A, B are two matrices such that A + B = 2 4  , A – B =  2 0 then AB
   
equals -

 2 4   2 4   2 4
(A) I (B)  4 4  (C)  3 2  (D)  4 4
     

 cos  sin  
9. If A =   sin  cos   , then A2 equals -
 

 cos 2 sin 2  cos 2  sin 2 


(A)   sin 2 cos 2  (B)  sin 2 cos 2 
 

 sin 2 cos 2  1 0 
(C)   cos 2 sin 2  (D) 0 1 
 

Page # 66
MATRICES
10. For matrices A and B, AB = 0, then -
(A) A = 0 or B = 0
(B) A = 0 and B = 0
(C) It is not necessary that A = 0 or B = 0
(D) All above statements are wrong

 1  tan  /2  tan  /2 1 


11.  tan  / 2 1   1 0  equals-

(A) zero matrix (B) sec 2  .I2


(C) I2 (D) None of these

0  i  0 1  1 0 
12. If A =  i 0  , B = 1 0 , C = 0 1 , then which of the following statement
     
is true-
(A) AB = BA (B) AB = – BA (C) A2 = BC (D) A2 = B + C

 2 1 
13. If A =  0 3 and f(x) = 2x2 – 3x, then f(A) equals -
 

14 1   14 1  14 1  14 1


(A)  0 9 (B)  0 9  (C)  0 9  (D)  0 9
   

2 0 1 
14. If A = 2 1 3 then A2 - 5A + 6I =
1 1 0 
 

1 1 5   1 1 3 
(A)  1 1 4  (B)  1 1 10 
 3 10 4   5 4 4 
  

(C) 0 (D) I

  
15. If     is square root of I 2, then  ,  and  will satisfy the relation -
 

(A) 1+  2 +  = 0 (B) 1–  2 +  = 0

(C) 1+  2 –  = 0 (D) –1+  2 +  = 0


16. If A and B are matrices of order m × n and n × m respectively, then the order
of matrix BT (AT)T is -
(A) m × n (B) m × m (C) n × n (D) Not defined

a b 
17. If A =  b a  , then |A +AT| equals -
 
(A) 4(a2 – b2) (B) 2(a2 – b2) (C) a2 – b2 (D) 4 ab
18. For suitable matrices A, B; the false statement is-
(A) (AB)T =ATB T (B) (AT)T= A
(C) (A – B)T =AT – B T (D) (AT)–1 = (A–1 )T

Page # 67
MATRICES

 cos  sin  
19. If A =   sin  cos   , then AA ' equals -
 
(A) I (B) A (C) A ' (D) 0
20. Let A be a square matrix. Then which of the following is not a symmetric matrix
(A) A + A' (B) AA ' (C) A ' A (D) A– A '

3 0 
21. For any 2 x 2 matrix A , A(adj A) = 0 3 , then |A| equals -
 
(A) 0 (B) 3 (C) 6 (D) 9

1 2 3 
 
22. If A = 2 3 4  , then the value of adj (adj A) is -
 0 0 2 

(A) |A|2 (B) –2A (C) 2 (D) A2

1 2 3 
23. If A = 0 3 1 , then A (adj A) equals -
2 1 2
 

9 0 0  9 0 0  0 0 9 
0 9 0     
(A) 0 0 9  (B) – 0 9 0  (C) 0 9 0  (D) None of these
  0 0 9  9 0 0 

 1 2 3 
 
24. If A =  4 0 1 , then (adj A)23 is equal to -
 3 1 5 

(A) 13 (B) –13 (C) 5 (D) –5


25. (adj AT ) – (adj A)T equals -
(A) 2|A| (B) 2|A|I (C) zero matrix (D) Unit matrix
26. The adjoint of symmetric matrix is -
(A) symmetric matrix
(B) skew-symmetric matrix
(C) diagonal matrix
(D) None of these

  1 4 
 
27. Matrix  3 0 1  is not invertible if -
 1 1 2 

(A)  = –15 (B)  = –17 (C)  = –16 (D)  = –18

1 2  1 0 
28. If A = 3 5  , B = 0 2 and X is a matrix such that A = BX, then X equals
   

1  2 4 1 2 4  2 4 
(A) (B) (C) 3 5  (D) None of these
2  3 5  2 3 5   

Page # 68
MATRICES

2 0 0 
29. If A = 0 2 0  , then value of A –1 is -
0 0 2 
 

1 0 0  1/ 2 0 0   2 0 0 
 0 1/2 0 
(A) 0 1 0  (B)  (C)  0 2 0  (D) None of these
0 0 1 
   0 0 1/ 2  0 0 2
 
30. For any square matrix A, which statement is wrong -
(A) (adj A)–1 = adj (A–1) (B) (AT)–1 = (A–1)T
(C) (A3)–1 = (A–1)3 (D) None of these

3 4
31 If A = 1 1 , then for every positive integer n, A n is equal to -
 

1  2n 4n  1  2n 4n 
(A)  n 1  2n  (B)  n 1  2n 

1  2n 4n 
(C)  n 1  2n  (D) None of these

32. If D = diag (d1, d2,.....,dn), then Dn equals -


(A) D (B) diag (d1n–1, d2n–1 , ....dnn–1 )
(C) diag (d1n, d2n, .....,dnn) (D) None of these

cos   sin  0 
 
33. If A =  sin  cos  0  , then -
 0 0 1

(A) adj A = A (B) adj A = A–1 (C) A–1 = –A (D) None of these

 cos  sin  
34. If A  =   sin  cos   , then which of following statement is true -
 

 cos n  sinn  
(A) A  . A = A  and ( A  )n =   sinn  cos n  
 

 cos n sin n 
(B) A  . A = A  and ( A  )n =   sin n cos n 
 

 cos n  sinn  
(C) A  . A = A  and ( A  )n =   sinn  cos n  
 

 cos n sin n 
(D) A  . A = A  and ( A  )n =   sin n cos n 
 
35. If A and B are square matrices such that AB = B and BA = A, then A 2 + B 2
is equal to -
(A) 2AB (B) 2BA (C) A + B (D) AB

Page # 69
MATRICES

 1 2 
36. If A be a matrix such that inverse of 7A is the matrix  4 7  , then A equals
 

 1 2  1 4 / 7 1 4  1 2/ 7 
(A)  4 1 (B) 2 /7 1/7  (C) 2 1  (D)  4 / 7 1/ 7 
     
37. If A and B are non- zero square matrices of the same order such that AB = 0,
then -
(A) adj A = 0 or adj B = 0 (B) adj A = 0 and adj B = 0
(C) |A| = 0 or |B| = 0 (D) None of these

 0 1
38. If A =  1 0  and (aI2 + bA)2 = A, then -
 

(A) a = b = 2 (B) a = b = 1/ 2 (C) a = b = 3 (D) a = b = 1/ 3

 0 c b   a 2 ab ac 
  ab b2 bc 
39. If A =  c 0 a  and B =  2
 , then AB is equal to -
 b a 0   ac bc c 

(A) A (B) B (C) I (D) 0


1
 1  tan    1 tan  a b 
40. If  tan  1    =  b a  , then -
    tan  1 

(A) a = 1, b = 1 (B) a = cos 2  , b = sin 2 


(C) a = sin 2  . b = cos 2  (D) None of these

1 3 
41. If A= 3 4 and A2 – kA – 5 I2 = O, then the value of k is-
 
(A) 3 (B) 5 (C) 7 (D) –7
cos   sin  0 
 
42. Let F (  )=  sin  cos  0  , where a  R. Then (F(  )) –1 is equal to
 0 0 1
(A) F (–  ) (B) F(  –1 ) (C) F(2  ) (D) none of these
43. If a matrix A is such that 3A + 2A + 5A + I = 0, then A–1 is equal to-
3 2

(A) –(3 A2 +2 A + 5) (B) (3 A2 +2 A + 5)


(C) 3A2 – 2 A – 5 (D) none of these

1 2 3  x   4 2
44. If 3 1 2  y    0 6 2 , then (x,y,z) is equal to-
     1 
2 3 1   z  1 2 
(A) (–4,2,2) (B) (4,–2,–2) (C) (4,2,2) (D) (–4,–2,–2)

1 0 1  a11 a12 a13 


45. If matrix A = 3 4 5  and its inverse is denoted by A 1  a 21 a 22 a 23  , then
0 6 7   
  a 31 a 32 a 33 
the value of a 23 is equal to-

21 1 2 2
(A) (B) (C)  (D)
20 5 5 5

Page # 70
MATRICES


a i bi 
46. If Ai =  i
a i 
and if |a|<1, |b|<1, then  det (Ai) is equal to-
b i 1

a2 b2 a 2  b2
(A) 2
 2 (B)
1  a  1  b  (1  a 2 )(1  b2 )

a2 b2 a b
(C) 2
 2 (D) 
1  a  1  b  1 a 1 b

1 1 1 
  adjB
47. If A= 0 2 3  and B = (adj A), and C = 5A, then is equal to-
C
2 1 0 

(A) 5 (B) 25 (C) –1 (D) 1

 1 2 0 2 1 5 
   
48. Let A + 2B =  6 3 3  and 2A – B = 2 1 6
 5 3 1   0 1 2 
then Tr (A) – Tr (B) has the value equal to
(A) 0 (B) 1 (C) 2 (D) none

 0 a b
1 1
49. If A=   a 0 c  , if Q1 = (A  A ') & Q2 = (A  A ') . Then Q1.Q2 is equal to :
2 2
  b  c 0 

(A) I3 (B) O 3 (C) A (D) A 2

 cos  sin  
50. If f(a) =   sin  cos   and if , ,  are angles of a triangle, then f(  ). f(  ). f(  ) =
 
(A) I2 (B) –I2 (C) 0 (D) None of these

cos   sin  0   cos  0 sin  


 sin  cos  0   0 
51. If M (  ) =   , M ( ) =  0 1
then
 0 0 1    sin  0 cos 

[M(  ) M (  )]–1 is equals to -


(A) M () M ( ) (B) M (–  ) M (–  )
(C) M (–  ) M(–  ) (D) –M (  ) M (  )

a b c 
52. If matrix A  b c a  , where a.b. c are real positive number, abc = 1 and A T A  I,
 c a b
 
then which of the following is/are true.
(A) a + b + c =5 (B) a2 + b2 + c2 = 6
(C) ab + bc+ ca = 2 (D) a3 + b3 + c3 = 4

Page # 71
MATRICES

1 5 1 5
53. If i  1 , a  , b then which of the following matrix is idempotent
2 2

a i  b i  a i  a b 
(A) i (B)  (C)  (D) 
 b  
 i a

 i b

b a 

 1 2
54. If A   6
 and if A = KA – 205I then
 1 3 
(A) K = 11 (B) K = 22 (C) K = 33 (D) K = 44
55. Let A is a 3 × 3 matrix and A = [aij]3 × 3. If for every column matrix X, if X1.A.X = 0 and
a23 = – 2009 then a32 = ........
(A) 2009 (B) –2009 (C) 0 (D) 2008

MULTIPLE CORRECT TYPE QUESTIONS


1. If AB = A and BA = B, then
(A) A2B = A2 (B) B2A = B2 (C) ABA = A (D) BAB = B

0 0 1 
2. If A  0 1 0  , then :
 
1 0 0 

 0 0 1
(A) Adj A is a zero matrix (B) Adj A   0 1 0 
 
1 0 0 
(C) A-1 = A 2
(D) A = I
3 If A and B are square matrices of the same order, then which of the following are
not true ?

(A)  
 AB   A B (B)  AB   BA 

(C) AB = 0 If |A| = 0 or |B| = 0 (D) AB = 0 If A = I or B =I

1 2 2
 
4. Let A = 2 1 2 , then
2 2 1

1
(A) A2 – 4A – 5I3 = 0 (B) A–1 = (A – 4I3)
5
(C) A3 is not invertible (D) A2 is invertible

1 1 1
5. If A  1 1 1 then
1 1 1
 

(A) A3 = 9A (B) A3 = 27A (C) A + A = A2 (D) A–1 does not exist

Page # 72
MATRICES
6. The value s of  and  for which the syste m of eq uations.
x  y  z  6,x  2y  3z  10,x  2y  z   have no solution are

(A) 3 (B)   10 (C)   3 (D)   10


7. A value of c for which the system of equations
x  y 1
 c  2 x   c  4  y  6
is solvable (consistent) is
 c  22 x   c  4 2 y  36
(A) 1 (B) 2 (C) 4 (D) None of these

0 2  
   
8. If the matrix   is orthogonal then
   

1 1 1
(A)  (B)    (C)    (D)    3
2 3 6
9. Suppose a1, a2, ....... real numbers, with a1  0. If a1, a2, a3, ..........are in A.P. then

 a1 a2 a3 
 a 6  is singular
(A) A = a 4 a5
a 5 a6 a 7 

(B) the system of equations a1x + a2y + a3z = 0, a4x + a5y + a6z = 0, a7x + a8y + a9z = 0
has infinite number of solutions

 a1 ia 2 
(C) B = ia a1  is non singular ; where i = 1
 2
(D) none of these
10. If D1 and D2 are two 3 x 3 diagonal matrices, then
(A) D1D2 is a diagonal matrix (B) D1D2 = D2D1
(C) D12 + D22 is a diagonal matrix (D) none of these

a b 
11. If A =  c d  (where bc  0) satisfies the equations x2 + k = 0, then
 
(A) a + d = 0 (B) k = –|A| (C) k = |A| (D) none of these

1 2 2
 
12. Let A = 2 1 2 , then
 2 2 1 

1
(A) A2 – 4A – 5I3 = 0 (B) A–1 = (A – 4I3)
5
(C) A3 is not invertible (D) A2 is invertible

Page # 73
MATRICES

x

1 x 3  16x ln (1  sin x )
13. Let a = Lim ; b = Lim ; c = Lim and
x 1 ln x x ln x x 0 4 x  x 2 x 0 x

( x  1) 3 a b 
d = Lim , then the matrix  c d  is
x 1 3sin( x  1)  ( x  1)   
(A) Idempotent (B) Involutary (C) Singular (D) Nilpotent
14. Let A be a square matrix of order 3 such that |AdjA| = 100, then |A| equals
(A)10 (B) - 10 (C)100 (D)25

COMPREHENSION TYPE QUESTIONS


I. For a given square matrix A, if there exists a matrix B such that AB = BA = I, then B
is called inverse of A. Every non–singular square matrix possess inverse and it
exists if |A|  0.

adj(A)
A–1 =  adj A = |A| (A–1).
det (A)

2 3 
1. Let, a matrix A =   , then it will satisfy the equation
1 2
(A) A2 – 4A + I = 0 (B) A2 + 4A + I = 0
(C) A2 – 4A – 5I = 0 (D) A2 – 4A + 5I = 0

2 3 
2. Let, a matrix A =   , then A–1 will be
1 2

 2 3  3 2  1 2  2 3
(A)  1 2  (B)  (C)  (D) 
  2 1 2 3 1 2

3 2 
3. Let matrix A =   satisfies the equation A2 + aA + bI = 0, then the value of
1 1 
4b
3
x
a
cosxdx equals

ab a  2b a  4b a  4b
(A) (B) (C) (D)
ab ab 4a  b 4a  b
II. Let
a11x1  a12 x 2  ..............a1n x n  b1;
a 21 x1  a 22 x 2  ..............a 2n x n  b2 ;
a n1 x1  a n2 x 2  ...........a nm x n  b n
be a system of n linear equations in n unknowns. Then this canbe
written in the matrix form as

Page # 74
MATRICES

 x1   b1 
 x2   b2 
   
 x3   b3 
[a
AX = B Where A = ij ]m  n; X  .
  B  .  Then
.  . 
.  . 
   
 xn   bn 

(I) If |A|  0, the system is consistent, and has a unique solution given by
X = A–1B.
(II) If |A| =0 and (adj A) B =0, then the system is consistent and has infinitely
many solutions.
(III) If (A) = 0 and (adj A ) B  0, then the system is inconsistent.
4. The system of equations 2x  y  3z  1, x  y  2z  5, x  y  z  1 has
(A) a unique solution (B) infinitely many solutions
(C) no solutions (D) finite number of solutions.
5 Let 2x  y  z  4, x  3y  2z  12,3x  2y  kz  10 . The value of k in the above sys-
tem of equations so that system does not have a unique solution is
(A) 2 (B) 3 (C) –1 (D) –2
6. If x  y  z  6, x  2y  3z  10, x  2y  z   , the values of  and  , for which the
system has infinitely many solutions is
(A)   3,   9 (B)   3,   10 (C)   2,   10 (D)   10,   3

MATRIX - MATCH TYPE QUESTION

1. Let A   1 tan x 
  tan x 1 

COLUMN-I COLUMN–II

 1 tan x 
(A) A 1 (p)   tan x 1 

 1  tan x 
(B)  adjA 1 (q) 2  tan x 1 

1 1  cos 2x  sin x 
(C) adj(adj A) (r)
2  sin 2x 1  cos 2x 

1  cos x sin 2x 
(D) adj (2A) (s)   sin 2x 1  cos 2x 
 

INTEGER TYPE QUESTIONS


1. 2x  3y  3z  0
5x  2y  2z  19
Find the value of x  y  z
x  7y  5z  5

Page # 75
MATRICES
2. For what value of 2k/33 the equations x  ky  3z  0,3x  ky  2z  0, 2x  3y  4z  0
possess a nontrival solution over the set of rationals ?

3  x 2 2 
 1  . Find the non zero value of x for which A 1 cannot be
3. Let A   2 4x

 2  4  1  x 

found .

0 2y z
4. Let A   x y  z  such that ATA = 1. Find the value of x2 + y2 + z2.
 x y z 

1   1 1 1 
5. A is a 2 × 2 matrix such that A  1 =  2  and A2  1 = 0 . The sum of the
       
elements of A, is

6. Number of value of 'a' for which the system of equations,


a2 x + (2 - a) y = 4 + a2
a x + (2 a - 1) y = a5 - 2 possess no solution is
x2 6 8 2x 3 5 
   2 2y
7. Let A =  3 y2 9, B =  6  . If trace A = trace B then x + y + z is equal
4 5 z 2   1 4 2z  3 

to
t 2  3t  4
8. If t is real and  = , then number of solutions of the system of
t2  3t  4
equations 3x - y + 4z = 3, x+ 2y - 3z = -2 and 6x + 5y +  z = -3 is

9. If A is a square matrix of order n such that |adj (adj A)| = |A|9, then the value of n
can be
10. Let a,b,c be positive real numbers. Then number of solutions of the system of equations
in x, y and z,
x2 y2 z x2 y2 z x2 y2 z 
   1 ,    1,     1 , is
a 2 b2 c2 a 2 b2 c2 a 2 b2 c 2

PREVIOUS YEARS QUESTIONS (AIEEE / JEE MAINS)


i 0 0 1 0 i 
1. If A = 0 i  , B = 1 0  and C =  i 0 , then A2 = B2 =C2 = [AIEEE 2002]
     
(A) I2 (B) I (C) –I (D) 2I

1 2 3   5 7 1 
3 1 2   
2. If A =   , B =  1 5 7  then AB = [AIEEE 2002]
2 3 1   7 1 5 

(A) I3 (B) 2 I3 (C) 4 I3 (D) 18 I3

Page # 76
MATRICES

a b    
3. If A =  b a  and A2 =     , then [AIEEE 2003]
   

(A)  = 2ab,  = a2 + b2 (B)  = a2 + b2,  = ab

(C)  = a2 + b2,  = 2ab (D)  = a2 + b2,  = a2 – b2

 0 0 1
4. Let A =  0 1 0  . The only correct statement about the matrix A is-
 1 0 0 
 

[AIEEE 2004]
(A) A is a zero matrix (B) A = (– 1) I, where I is a unit matrix
(C) A–1 does not exist (D) A2 = I

 1 1 1   4 2 2
 2 1 3   
5. Let A =   and (10)B =  5 0   . If B is the inverse of matrix A, then a
1 1 1   1 2 3 
is- [AIEEE 2004]
(A) –2 (B) –1 (C) 2 (D) 5
6. If A2 – A + I = 0, then the inverse of A is - [AIEEE-2005]
(A) A+I (B) A (C) A – I (D) I – A

1 0  1 0 
7. If A = 1 1  and I = 0 1  , then which one of the following holds for all n  1, by
   
the principle of mathematical induction - [AIEEE-2005]
(A) An = nA – (n – 1) I (B) An = 2n–1 A – (n – 1) I
(C) An = nA + (n – I) I (D) An = 2n–1A + (n – 1) I
8. If A and B are square matrices of size n × n such that A2 – B2 = (A – B) (A + B), then
which of the following will be always true – [AIEEE 2006]
(A) AB = BA
(B) Either of A or B is a zero matrix
(C) Either of A or B is an identity matrix
(D) A = B

1 2 a 0
9. Let A =  3 4  and B =  0 b  , a, b  N. Then – [AIEEE 2006]
   
(A) there exist more than one but finite number of B's such that AB = BA
(B) there exist exactly one B such that AB = BA
(C) there exist infinitely many B's such that AB = BA
(D) there cannot exist any B such that AB = BA

5 5  
 
10. Let A = 0  5  If |A2| = 25, then |  | equals- [AIEEE 2007]
0 0 5

1
(A) 52 (B) 1 (C) (D) 5
5

Page # 77
MATRICES
11. Let A be a square matrix all of whose entries are integers. Then which one of the
following is true ? [AIEEE 2008]

(A) If det A  ± 1, then A–1 exists and all its entries are non-integers

(B) If det A = ± 1, then A–1 exists and all its entries are integers

(C) If det A = ± 1, then A–1 need not exist

(D) If det A = ± 1, then A–1 exists but all its entries are not necessarily integers
Directions : Question number 12-13 is Assertion-Reason type question. This question
contains two statements: Statement-1 (Assertion) and Statement-2 (reason).This
questions also has four alternative choices, only one of which is the correct answer.
You have to select the correct choice.
12. Let A be a 2 × 2 matrix with real entries. Let I be the 2 × 2 identity matrix. Denote by
tr (A), the sum of diagonal entries of A, Assume that A2 = I. [AIEEE 2008]
Statement- 1: If A  I and A  – I, then det A = – 1
Statement -2 : If A  I and A  –I, then tr (A)  0
(A) Statement-1 is true , Statement -2 is true; Sta te ment-2 is a corre ct
explanation for Statement-1
(B) Statement-1 is true, Statement -2 is true; Statement-2 is not a correct
explanation for Statement-1
(C) Statement-1 is true, Statement -2 is false
(D) Statement-1 is false, Statement-2 is true
13. Let A be a 2 × 2 matrix [AIEEE 2009]
Statement- 1: adj (adj A) = A
Statement -2 : | adj A | = | A |
(A) Statement-1 is true, Statement -2 is true; Statement-2 is not a correct
explanation for Statement-1
(B) Statement-1 is true, Statement -2 is false
(C) Statement-1 is false, Statement -2 is true
(D) Statement-1 is true, Statement-2 is true, Statement-2 is a correct explanation
for Statement-1
14. Let A and B be two symmetric matrices of order 3. [AIEEE 2011]
Statement-1: A (BA) and (AB) A are symmetric matrices
Statement-2: AB is symmetric matrix if matrix multiplication of A and B is
commutative.
(A) Statement-1 is true, Statement-2 is true; Statement-2 is not a correct
explanation for Statement-1
(B) Statement-1is true, Statement-2 is false
(C) Statement-1 is false, Statement-2 is true
(D) Statement-1 is true, Statement-2 is true; Statement-2 is a correct explanation
for Statement-1

Page # 78
MATRICES

1 0 0 1 
 2 1 0  
15. Let A =   . If u1 and u2 are column matrices matrices such that Au1 =  0  and
3 2 1 0

0
 
Au2 = 1  , then u1 + u2 is equal to [AIEEE 2012]
0

 1   1  1   1
 1      
(A)  0 (B)  1  (C)  1  (D)  1
   1  0  1

1  3
16. If P  1 3 3  is the adjoint of a 3 × 3 matrix A and |A| = 4, then  is equal to
 
2 4 4

[Mains-2013]
(A) 11 (B) 5 (C) 0 (D) 4

17. If A is an 3 × 3 non singular matrix such that AA = A ' A and B  A 1A , then
BB equals [Mains-2014]

(A) B –1 (B) (B1 ) (C) I + B (D) I

1 2 2 
18. If A   2 1 2 is a matrix satisfying the equation AAT = 9I, where I is 3×3 identity
a 2 b 

matrix, then the ordered pair (a, b) is equal to : [JEE MAINS 2015]
(A) (2, –1) (B) (–2, 1) (C) (2, 1) (D) (–2, –1)

5a b 
19. If A   and A adj A = A AT, then 5a + b is equal to: [JEE MAINS 2016]
3 2 

(A) 5 (B) 4 (C) 13 (D) –1

PREVIOUS YEARS QUESTIONS (IIT JEE / JEE ADVANCED)


Single Correct Questions:
1. If the system of equations x  ky  z  0,kx  y  z  0, x  y  z  0 has non zero solution,
then possible values of k are [IIT-2000]
(A) 1, 2 (B) 1, 2 (C) 0,1 (D) -1,1

 0  1 0  2
2. If A    and B    , then the value of  for which A = B is [IIT-2003]
1 1  5 1 
(A) 1 (B) –1 (C) 4 (D) no real values.

Page # 79
MATRICES

 2  3
3. If A    and|A | 125 , then the value of  is : [IIT-2004]
 2  
(A) 1 (B) 2 (C) 3 (D) 5
4. The number of 3 × 3 matrices A whose entries are either 0 or 1 and for which the

 x  1 
   
system A  y   0  has exactly two distinct solutions, is [IIT 2010]
 z  0 

(A) 0 (B) 29 – 1 (C) 168 (D) 2


5. Let M and N be two 3 × 3 non-singular skew symmetric matrices such that MN =
NM. If PT denotes the transpose of P, then M2N2 (MTN)–1 (MN–1)T is equal to
[IIT 2011]
(A) M2 (B) –N 2 (C) –M2 (D) MN
6. Let   1 be a cube root of unity and S be the set of all non-singular matrices of

1 a b
 1 c  , where each of a, b, and c is either  or  2. Then the number
the form  
2  1 
of distinct matrices in the set S is [IIT 2011]
(A) 2 (B) 6 (C) 4 (D) 8
T T
7. If P is a 3 × 3 matrix such that P = 2P + I, where P is the transpose of P and I is the

 x  0
   
3 × 3 identity matrix, then there exists a column matrix X  y  0 such that
   
 z  0
[IIT 2012]

 0
(A) PX  0 (B) PX = X (C) PX = 2X (D) PX = - X
0

 1 0 0
 
8. Let P   4 1 0 and I be the identity matrix of order 3. If Q  [q ij ] is a matrix such
16 4 1 

q 31  q 32
that p50  Q  I , then q 21
equals [JEE ADV 2016]

(A) 52 (B) 103 (C) 201 (D) 205

Page # 80
MATRICES

More than one correct:


9. For 3 × 3 matrices M and N, which of the following statement(s) is (are) NOT
correct? [JEE Adv. - 2013]
(A) NTMN is symmetric or skew symmetric, according as M is symmetric or skew
symmetric
(B) MN - NM is skew symmetric for all symmetric matrices M and N
(C) MN is symmetric for all symmetric matrices M and N
(D) (adj M) (adj N) = adj(MN) for all invertible matrices M and N

10. Let  be a complex cube root of unity with   1 and P  Pij  be a n × n matrix with

pij  i  j . then P2  0 when n = [JEE Adv. 2013]

(A) 57 (B) 55 (C) 58 (D) 56


11. Let X and Y be two arbitrary, 3 × 3, non zero, skew symmetric matrices and Z be an
arbitrary 3 × 3 non zero, symmetric matric matrix. Then which of the following
matrices is(are) skew symmetric ? [JEE Adv. 2015]
(A) Y3Z4 - Z4Y3 (B) X44 + Y44 (C) X4Z3 – Z3X 4 (D) X23 + Y23

 3 1 2 
12. Let P  2 0   , where   R . Suppose Q = [qij] is a matrix such that PQ = kI,
3 5 0 

k k2
where k , k  R and I is the identity matrix of order 3. If q 23   and det(Q)  ,then
8 2
[JEE Adv. 2016]
(A)   0, k = 8 (B) 4  k  8  0

(C) det(P adj(Q))  29 (D) det(Q adj(P))  213

Comprehension:

1 0 0 
I. A  2 1 0  . if U1, U2 and U3 are column matrices satisfying
3 2 1 
 

1   2  2
AU1  0  , AU2  3 and AU3  3 ,U
0  0  1  is 3 3 matrix whose columns are U1,U2 ,U3
     
then answer the following questions [IIT 2006]

13. The value of U is


(A) 3 (B) -3 (C) 3/2 (D) 2

Page # 81
MATRICES
14. The sum of the elements of U1 is
(A) -1 (B) 0 (C) 1 (D) 3

3 
 
15. The value of [3 2 0] U 2 is
0 

5  3 
(A) [5] (B)   (C) [4] (D)  
2 2
II. Let p be an odd prime number and Tp be the following set of 2 × 2 matrices.

 a b  
Tp =  A   c a  :a,b,c  {0, 1, 2, ...,p  1}  [IIT 2010]
   
16. The number of A in Tp such that A is either symmetric or skew–symmetric or both,
and det(A) divisible by p is
(A) (p – 1)2 (B) 2 (p – 1)
(C) (p – 1)2 + 1 (D) 2p – 1
17. The number of A in Tp such that the trace of A is not divisible by p but det (A) is
divisible by p is [Note: The trace of a matrix is the sum of its diagonal entries.]
(A) (p – q) (p2 - p + 1) (B) p3 – (p – 1)2
(C) (p – 1)2 (D) (p – 1) (p2 – 2)
18. The number of A in Tp such that det (A) is not divisible by p is
(A) 2p2 (B) p3 – 5p
(C) p3 – 3p (D) p3 – p2

Subjective Questions

a b c 
19. If matrix A  b c a  , where a, b, c are real positive numbers, abc = 1 and AT A = I,
 
 c a b 

then find the value of a3 + b3 + c3. [IIT-2003]

20. If M is a 3  3 matrix , where M T M  I and det  M   1 then prove that det  M  I   0


[IIT-2004]

a 0 1  a 1 1  f  a 2 
21. A  1 c b  , B  0 d c  , U 
  g  , V   0  [IIT-2004]
   
1 d b   f g h h 0
 

If there is a vector matrix X, such that AX = U has infinitely many solutions, then

Page # 82
MATRICES
prove that BX = V cannot have a unique solution. If afd  0. Then prove that BX = V
has no solution.
Integer Questions:
22. Let k be a positive real number and let

2k  1 2 k 2 k  0 2k  1 k 
   
A= 2 k 1 2k  and B = 1  2k 0 2 k.
   
 2 k 2k 1    k 2 k 0 

If det (adj A) + det (adj B) = 106, then [k] is equal to

{Note: adj M denotes the adjoint of a square matrix M and [k] denotes the largest
integer less than or equal to k]. [IIT-2010]
23. Let M be a 3×3 matrix satisfying

 0   1   1   1 1  0 
           
M 1    2 , M  1    1 , and M 1   0  .
0   3  0   1 1 12

Then the sum of the diagonal entries of M is [IIT-2011]

1  3i  r
z 2s 
24. Let z  , where i  1 , and r,s  {1,2,3} .Let P  (z)  and I be the
2 2s
zr 
z
identity marrix of order 2. Then the total number of ordered pairs (r,s) for which P 2  I
is [JEE ADV 2016]

Page # 83
MATRICES

ANSWER KEY
DRILL - I
1. n = -3, y = -5, z = +2. a = -2, b = -7, c = -1

 11 / 2 9 7 / 2 
2. 512 3. diag [-11, 15, 23] 4.  13 / 2 0 7 / 2 

 1 8 2 
 5 6 13  7 22 
7.  5 3 16  8. (i) 0, (ii) 0 9.  33 31 
 

 3 3 / 2   0 5 / 2
11. a = 4, b = 2, c  R, d = -3 12.  3 / 2 ,
1   5 / 2 0 

1 1 1
13.  ,b=  ,c= 
2 6 3
DRILL - II

1 / 2 1 / 2 1 / 2 
 5 2   4 3 1 
1.  7 3  2. 5 / 2 3 / 2 1 / 2  5. 1024
 
6. -1029 7. 1/6 8. 1
 5 11 9 
 
9. 0 10.  1 2 2  11. x = 15000, y = 15000
 3 6 5 

 3 0 0
1  1  5 3 
13. 3  3 1 0
 14. 15. 25
9 2 3 14  2 4 
 
16. 25 17. 256
5  3k
20. Consistent, Infinite solutions, x = , y=k, k  R
4
21. Consistent, unique solution
17  5k 1  11k
22. Consistent, x = , y= , z=k
7 7
23. x = 1, y =2, z = 3 24. x = 2, y = -1, z = 4
25. x = 1, y = 2, z = 3 26. x = 5, y =8, z =8

Page # 84
MATRICES
SINGLE CORRECT TYPE QUESTIONS
1. B 2. C 3. C 4. D 5. B 6. C

7. A 8. B 9. A 10. C 11. D 12. B

13. C 14. B 15. D 16. D 17. A 18. A


19. A 20. D 21. B 22. B 23. B 24. A

25. C 26. A 27. B 28. B 29. B 30. D

31. B 32. C 33. B 34. D 35. C 36. D


37. C 38. B 39. D 40. B 41. B 42. A

43. D 44. A 45. D 46. B 47. D 48. C


49. B 50. B 51. C 52. D 53. C 54. D

55. A

MULTIPLE CORRECT TYPE QUESTIONS


1. ABCD 2. BCD 3. AD 4. ABD 5. AD 6. AD

7. BC 8. ACD 9. ABC 10. ABC 11. AC 12. ABD

13. CD 14. AB
COMPREHENSION TYPE QUESTIONS

1. A 2. A 3. C 4. A 5. B 6. B

MATRIX - MATCH TYPE QUESTIONS


1. (A  R) ;(B  S); (C  P);(D  Q)

INTEGER TYPE QUESTIONS

1. (1) 2. (1) 3. (3) 4. (1) 5. (5) 6. (2)


7. (3) 8. (1) 9. (4) 10. (8)

PREVIOUS YEAR QUESTIONS (AIEEE)


1. C 2. D 3. C 4. D 5. D 6. D

7. A 8. A 9. C 10. C 11. B 12. C

13. A 14. A 15. D 16. A 17. D 18. D


19. A

PREVIOUS YEAR QUESTIONS (IIT JEE)

1. D 2. D 3. C 4. A 5. C 6. A
7. D 8. B 9. C, D 10. BCD 11. CD 12. BC

13. A 14. B 15. A 16. D 17. C 18. D

19. 4 22. 4 23. 9 24. 1

Page # 85

You might also like